Skin Lesions Flashcards

1
Q

An obese 47-year-old man comes to the office for pain and drainage in the perineal region. A photograph is shown. Medical history includes hypertension, hypercholesterolemia, and diabetes. Which of the following is the most important factor in reducing the risk of recurrence after surgery?

A) Adjuvant radiation
B) Antibiotic therapy
C) Extent of resection
D) Intralesional corticosteroid injection
E) Method of closure

A

The correct response is Option C.

This patient has hidradenitis suppurative (HS). There are various surgical treatments available to these patients with varying risk of recurrence. Surgical options include incision and drainage, surgical deroofing, local excision, and radical resection of all involved tissue. Reconstructive and closure techniques include secondary healing, primary closure, skin grafting and locoregional pedicled flaps. Numerous studies have looked at the outcomes with various closure techniques and they show that risk of recurrence is likely influenced more by the extent of disease rather than the method of definitive closure. Because of the recurrent nature of this disease, surgery has been considered as the only effective curative therapy for HS. Inadvertent compromise in the margin of resection may diminish the probability of successful healing. Radical resection of all hair-bearing skin with a 1- to 2-cm clear margin of normal tissue is the gold standard and the most important factor in reducing risk of recurrence. Recurrence rates tend to be higher after excision in regions where functional and aesthetic outcomes take priority, and often limit the extent of resection and compromise the ability to obtain clear margins (ie, axilla, perineum, breast). Historically, low-voltage radiation was used as a treatment modality for HS and it is thought to cause complete follicular destruction. Current studies on radiotherapy as treatment for HS are very limited because of concern regarding malignancies arising in radiation fields. Radiation should be reserved for individuals with severely recalcitrant disease and used with extreme caution in younger individuals. Bacterial burden may also diminish the probability of successful wound healing. Topical and systemic antibiotics are still the mainstay treatment for mild HS. Despite their widespread use, few studies have shown their efficacy. In addition, antibiotics do offer relief by reducing the burden of abscesses and pustules in some individuals, but recurrence in these people is frequent. Standard practice of managing acute flares with intra-lesional steroid injections lacks clinical evidence. It has been shown in a series of patients to reduce erythema, edema, size and pain, but no effect on recurrence of disease has been shown.

2018

How well did you know this?
1
Not at all
2
3
4
5
Perfectly
2
Q

A 50-year-old woman is evaluated for multiple firm, nodular, pink-colored lesions of the scalp, ranging in size from 2 to 4 mm. Examination of a specimen obtained on biopsy shows benign cylindroma. Which of the following is the most appropriate management of these lesions?

A) Cryotherapy
B) Electrodessication and curettage
C) Imiquimod therapy
D) Radiotherapy
E) Surgical excision

A

The correct response is Option E.

Cylindromas are benign adnexal tumors showing an eccrine and an apocrine differentiation. They are found most commonly on the scalp and face, and are more common in women. Solitary cylindromas are generally sporadic in nature. Multiple cylindromas can be seen in patients with Brooke-Spiegler syndrome as an autosomal dominant trait with variable penetrance.

Cylindromas may undergo malignant transformation, and therefore surgical excision is typically recommended, with close postoperative follow-up given high recurrence rates.

Cryotherapy, electrodessication and curettage, and imiquimod are not treatments for cylindromas.

Radiotherapy has been used to treat malignant cylindromas (also known as cylindrocarcinoma or adenoid cystic carcinoma), but not benign cylindromas.

2018

How well did you know this?
1
Not at all
2
3
4
5
Perfectly
3
Q

A 56-year-old woman presents with a 1-cm primary superficial basal cell carcinoma on the left volar mid-forearm. Medical history includes renal transplantation, carcinoma of the right breast managed with lumpectomy and radiation, and treatment for a gunshot wound to the left forearm. The basal cell carcinoma is located within the previous traumatic scar. Which of the following clinical features is an indication for Mohs micrographic surgery in this patient?

A) Basal cell carcinoma arising in traumatic scar
B) History of radiation
C) Immunocompromised status of patient
D) Size of basal cell carcinoma
E) Superficial basal cell carcinoma

A

The correct response is Option A.

The clinical feature in this particular patient that fulfills the criteria to get Mohs micrographic surgery is that the basal cell carcinoma is arising in a traumatic scar. There are many clear indications for Mohs micrographic surgery for basal cell carcinoma: certain size, histology, and anatomic location, all recurrent basal cell carcinomas, and the occurrence of basal cell carcinoma in irradiated skin, traumatic scars, areas with osteomyelitis/chronic ulceration/inflammation, and/or patients with genetic syndromes. Almost all primary basal cell carcinomas in the H and M zones, regardless of pathology (i.e. superficial, nodular, or aggressive), size, or health status of the patient, are candidates for Mohs micrographic surgery. In the L zone, most basal cell carcinomas are also candidates for Mohs micrographic surgery (except all superficial subtypes [irrespective of health of patient], or those less than 1 cm size in immunocompromised patients or nodular subtypes).

Area H: “Mask areas” of face (central face, eyelids [including inner/outer canthi], eyebrows, nose, lips [cutaneous/mucosal/vermilion], chin, ear and periauricular skin /sulci, temple), genitalia (including perineal and perianal), hands, feet, nail units, ankles, and nipples/areola.

Area M: Cheeks, forehead, scalp, neck, jawline, and pretibial surface.

Area L: Trunk and extremities (excluding pretibial surface, hands, feet, nail units and ankles).

In this clinical case, the patient has a basal cell carcinoma that has a favorable pathology (i.e. superficial subtype) in the L zone, not an indication for Mohs micrographic surgery, irrespective of tumor size. Additionally, with a 1-cm tumor of this pathology subtype, her immunocompromised state is not an indication for Mohs micrographic surgery, either. The radiation was remote from the area she developed her basal cell carcinoma, so it is not an indication, either.

2018

How well did you know this?
1
Not at all
2
3
4
5
Perfectly
4
Q

A 60-year-old man presents with a 6-mm lesion of the forehead. The patient states that it has enlarged over a period of 2 years. Examination of a biopsy specimen shows squamous cell carcinoma (adenoid subtype) with a 1.5-mm depth of involvement. Which of the following is the most likely risk factor for recurrence of this patient’s lesion after surgical excision?

A) Anatomic location
B) Depth of involvement
C) Growth rate
D) Histologic subtype
E) Size

A

The correct response is Option D.

A number of risk factors for recurrence have been identified for squamous cell carcinomas.

Histologic subtype is one such factor. Adenoid, adenosquamous, and desmoplastic subtypes are considered high risk for recurrence.

Anatomic location is another such factor and is typically considered in combination with the size of the lesion. High-risk areas include the “mask” areas of the face (eyelids, eyebrows, periorbital, nose, lips, chin, temple, ear), as well as genitalia, hands, and feet. Lesions greater than 6 mm in these areas indicate a high risk for recurrence. Mid-risk areas include the cheeks, forehead, scalp, and neck, with lesions greater than 10 mm indicating a high risk for recurrence. Low-risk areas include the trunk and extremities, with lesions greater than 20 mm indicating a high risk for recurrence. Based on this, this patient with a 6-mm forehead lesion would not be considered at high risk for recurrence.

Rapid growth is a risk factor for recurrence. However, this patient’s lesion grew gradually over a period of 2 years and not rapidly.

Depth of involvement greater than 2 mm indicates a high risk for recurrence.

Other risk factors for recurrence include poorly defined borders, immunosuppression, prior irradiation, site of inflammatory process, neurologic symptoms, moderate/poor differentiation, and perineural/vascular involvement.

2018

How well did you know this?
1
Not at all
2
3
4
5
Perfectly
5
Q

Which of the following is the most appropriate surgical treatment recommendation for a 4-cm round sebaceous nevus of the scalp in a child?

A) Excision, skin grafting, and delayed tissue expansion
B) Serial excision without tissue expansion
C) Serial monitoring and selective excision
D) Tissue expansion and excision
E) Tissue expansion, excision, and rotational flaps

A

The correct response is Option C.

Historically, there has been a 10 to 15% reported malignant degeneration in nevus sebaceous in children and the recommendation had been for all lesions to be removed before puberty. More recent studies have shown malignant transformation of these lesions to be less than 1%. Therefore, the decrease in malignant change prompted a more conservative approach to nevus sebaceous based on serial monitoring and excision of lesions that are a source of irritation, difficult to monitor clinically, or aesthetically displeasing. Tissue expansion is an option in patients with giant nevus sebaceous and usually requires one to two rounds of expansion and serial excisions. However, this is not a first line treatment for a small 4-cm lesion. Although serial excision is an alternative reconstructive option, it is recommended only for larger lesions that can be excised in three stages or less. If that is not possible, then tissue expansion is the preferred procedure. Reconstruction with rotational flaps should be planned in consideration of natural facial aesthetic units and an effort should be made to maintain natural brow and hairline position and symmetry and avoid creating tension in the perioral, periorbital, and periauricular regions.

2018

How well did you know this?
1
Not at all
2
3
4
5
Perfectly
6
Q

Which of the following peripheral nerve tumors is most commonly associated with von Recklinghausen disease?

A) Astrocytoma
B) Glioblastoma
C) Neurilemoma
D) Neurofibroma
E) Schwannoma

A

The correct response is Option D.

A neurofibroma is a lesion of the peripheral nervous system, which is derived from Schwann cells, other perineural cell lines, and fibroblasts. Neurofibromas may arise sporadically, or in association with von Recklinghausen disease (neurofibromatosis 1 or NF1). A neurofibroma may arise at any point along a peripheral nerve, and comes in two varieties.

The plexiform neurofibromas are larger tumors that develop inside the body and tend to intimately involve the nerves, blood vessels, and other structures in the body. They can reside deep inside the body or closer to the skin. Plexiform neurofibromas can cause pain, numbness, weakness, and disfigurement. These tumors do have a small chance of becoming cancerous. Plexiform neurofibromas may also be asymptomatic.

Dermal (subcutaneous) neurofibromas are small, nodule-like tumors that grow on or just under the surface of the skin. They can be painful, itchy, disfiguring, or tender when touched, but they have no known potential to become cancerous. Dermal neurofibromas may also be asymptomatic.

Schwannomas are peripheral nerve sheath tumors that can be seen with NF1, but are more commonly associated with neurofibromatosis 2. The major distinction between a schwannoma and a solitary neurofibroma is that a schwannoma can be resected while sparing the underlying nerve, whereas resection of a neurofibroma requires the sacrifice of the underlying nerve. A neurilemoma is another name for a schwannoma.

Astrocytomas and glioblastomas are tumors of the central nervous system. Astrocytomas and optic gliomas can be seen in association with NF1.

2018

How well did you know this?
1
Not at all
2
3
4
5
Perfectly
7
Q

A 13-year-old girl is brought for evaluation because of a 4-month history of severe pain of the tip of the right index finger. There is no history of trauma. On examination, the finger appears normal with no visible swelling or discoloration. The pain is exacerbated by local pressure when the patient writes and during her weekly swimming lessons. MRI (T2-weighted) image is shown. Which of the following is the most likely diagnosis?

A) Digital fibroma
B) Giant cell tumor
C) Glomus tumor
D) Neuroma
E) Venous malformation

A

The correct response is Option C.

This lesion is a glomus tumor. Glomus tumors are benign hamartomas originating from the glomus body, a structure comprised of vascular and neural elements that is responsible for thermoregulation in the skin. These often inconspicuous tumors present with pain, point tenderness, and sensitivity to cold. X-ray studies may show cortical erosion of the bone adjacent to the lesion, and ultrasonography can provide confirmation. MRI is the most accurate imaging modality and the tumor appears as a bright, discrete mass on T2-weighted images.

Digital fibroma is a cutaneous fibroblastic proliferation and would be visible. It rarely causes pain and would not enhance on T2-weighted MRI imaging. Neuroma can cause focal pain as described in the vignette, but the enhancing focal lesion on the T2-weighted MRI effectively rules out solid masses such as neuroma or giant cell tumor (which is common but rarely causes pain or cold intolerance). Venous malformation would enhance on T2-weighted MRI, but would typically present with swelling and would not be as well circumscribed as the lesion shown here.

2018

How well did you know this?
1
Not at all
2
3
4
5
Perfectly
8
Q

A 72-year-old man comes to the office for evaluation of a 2-cm growth over the lower half of his nose that has been growing slowly during the past 15 years. A photograph is shown. Medical history includes acne rosacea. Physical examination shows a broad, exophytic mass. Which of the following is the most appropriate management?

A) Application of 5-fluorouracil
B) Direct excision and coverage with a forehead flap
C) Direct excision and coverage with a skin graft
D) Direct excision and healing by secondary intention
E) Observation only

A

The correct response is Option D.

Rhinophyma occurs on the far end of progressive acne rosacea involving the sebaceous glandular overgrowth of the skin. It most commonly occurs on the nose but phymas can occur elsewhere on the face.

In early stages, rhinophyma can be treated with isoretinoin and antibiotics; however, in advanced states it needs to be resected. Ablation modalities include dermabrasion, carbon dioxide laser excision, scalpel excision, or a combination of techniques. Since the procedure resects the skin to mid-dermis, skin grafting or other coverage is unnecessary and can lead to unsatisfactory aesthetic results. The best option is excision with secondary epithelialization.

2018

How well did you know this?
1
Not at all
2
3
4
5
Perfectly
9
Q

An 85-year-old man who takes an anticoagulant medication comes to the office for evaluation of a recurrent 1-cm nodular basal cell carcinoma at his nasal tip that has started to bleed intermittently. Medical history includes placement of a cardiac stent 1 month ago, after myocardial infarction. Which of the following is the most appropriate treatment for this patient?

A) Electrodessication
B) Excision with forehead flap
C) Pembrolizumab therapy
D) Superficial radiation therapy
E) Topical application of 5-fluorouracil

A

The correct response is Option D.

With a 5-year recurrence rate of about 3% for nodular basal cell carcinomas (BCC), superficial radiation therapy has become a viable alternative to Mohs micrographic surgery, which remains the gold standard for treatment of nonmelanomatous skin cancers.

Electrodessication has a high recurrence rate and would subject this anti-coagulated patient to the risk of bleeding after surgery or thrombosis should his anticoagulation be discontinued.

Excision with forehead flap would not be safe for a patient with a recent myocardial infarction.

Topical 5-fluorouracil is not indicated in the management of nodular BCC.

Pembrolizumab therapy is indicated for Stage IV melanoma.

2018

How well did you know this?
1
Not at all
2
3
4
5
Perfectly
10
Q

A 34-year-old woman comes to the office because of a 6 × 7-cm subcutaneous mass below the left scapula. Biopsy confirms dermatofibrosarcoma protuberans. To minimize recurrence yet maximize the chances of primary closure, which of the following is the most appropriate margin when planning wide local excision?

A) 5 mm
B) 10 mm
C) 20 mm
D) 40 mm
E) 50 mm

A

The correct response is Option C.

Several recent studies have confirmed that a surgical margin of 15 to 20 mm is associated with high rates of recurrence-free survival and primary closure when wide local excision is performed. Marginal excision is associated with higher rates of recurrence, and larger wide local excisions (>20 mm) are associated with similar recurrence-free survival but a much higher need for reconstructive surgery. Mohs micrographic surgery has shown promise, with higher initial clearance rates using smaller margins, but the question specifically addressed surgical margins when planning wide local excision.

2017

How well did you know this?
1
Not at all
2
3
4
5
Perfectly
11
Q

A 12-year-old girl develops a 12-mm nodule on her right cheek that grows slowly over 2 months. It is firm to the touch, mildly tender, and slightly bluish. There is no redness, ulceration, or visible punctum. Which of the following is the most likely diagnosis?

A) Hemangioma
B) Keratinous cyst
C) Pilomatricoma
D) Sebaceous nevus
E) Spitz nevus

A

The correct response is Option C.

Pilomatricoma (also known as pilomatrixoma or calcifying epithelioma of Malherbe) is a common, benign calcifying tumor of the hair appendages that mostly occur under the age of 20. Most occur in the head and neck, but the extremities and trunk are also affected. Surgical excision is the treatment of choice. Malignancy is very rare. Intraoperative findings show a calcific, friable mass adherent to the undersurface of the skin. Unlike a keratinous or sebaceous cyst, there is no discrete capsule or punctum (plugged pore). Recurrence is reported in the 1 to 2% range. This benign growth is related to a somatic (non-inherited) gene mutation CTNNB-1, that is involved in cell replication of the hair matrix.

Sebaceous nevus presents as a waxy textured skin patch, often present at birth. The scalp is a common site, although it may present elsewhere. Lesions are slow-growing and benign, but over the course of one’s lifetime, they have up to a 50% transformation rate to basal cell carcinoma, with squamous cell carcinoma less likely.

Hemangiomas are cutaneous blood vessel proliferations that are bright red or purple in color and are typically present at birth. While they are also common in childhood, they are more superficial in location and have a very different appearance versus pilomatricomas, which are subepithelial.

Spitz nevi are melanocytic lesions that can occur in children and adults. They can mimic melanoma though they are benign spindle cell lesions. Malignant transformation is not common, though atypical variants exist, therefore excision is recommended. They appear as dark brown or black macules on the skin.

2017

How well did you know this?
1
Not at all
2
3
4
5
Perfectly
12
Q

A 58-year-old Caucasian farmer comes to the office because of several pink scaly macules on his cheeks and nose. Biopsy of one of the lesions shows pleomorphic keratinocytes within the basal layer of the epidermis and hyperkeratosis, consistent with actinic keratosis. If left untreated, which of the following is the likelihood that these lesions will become malignant?

A) 0%
B) 10%
C) 30%
D) 50%
E) 70%

A

The correct response is Option B.

The patient has actinic keratoses (AKs). AKs are common in in people with significant sun exposure and are a response to ultraviolet radiation. The likelihood of malignant transformation to squamous cell carcinoma (SCC) is approximately 10%.

There are various treatment modalities for AKs, including cryotherapy, 5-flurouracil (5-FU), photodynamic therapy, superficial glycolic peels, and imiquimod. Cryotherapy with liquid nitrogen is commonly used for isolated lesions, while the other therapies are more commonly used for diffuse disease.

2017

How well did you know this?
1
Not at all
2
3
4
5
Perfectly
13
Q

A 7-year-old boy is evaluated because of new nodular lesions on his skin. Patient history includes a jaw cyst, pits in the hands and feet, and a treated meduloblastoma. Biopsy is planned. Which of the following is the most likely diagnosis?

A) Basal cell carcinoma
B) Melanoma
C) Merkel cell carcinoma
D) Sebaceous adenocarcinoma
E) Squamous cell carcinoma

A

The correct response is Option A.

The patient has basal cell nevus (Gorlin) syndrome. It is an autosomal dominant genetic condition affecting 1 in 56,000. Males and females are equally affected. It is caused by a mutation in the PTCH1 gene. Clinical characteristics include multiple basal cell carcinomas, odontogenic cysts of the mandible, facial dysmorphism, and skeletal abnormalities of the vertebrae, skull, and ribs. 5 to 10% of patients will develop medulloblastoma. The other malignancies are not associated with Gorlin syndrome.

2017

How well did you know this?
1
Not at all
2
3
4
5
Perfectly
14
Q

A 50-year-old woman with a history of scleroderma is evaluated because of a 1.5-cm lesion on her right cheek. Patient history includes basal cell carcinoma excision at the same site 3 years ago. A punch biopsy shows basal cell carcinoma (micronodular subtype). Which of the following is the most appropriate indication for Mohs micrographic surgery in this patient?

A) Histologic subtype
B) History of scleroderma
C) Location of lesion
D) Recurrence of lesion
E) Size of lesion

A

The correct response is Option D.

Mohs micrographic surgery is a surgical technique in which tumor excision and microscopic examination of tissue margins are performed by the same surgeon. Use of a beveled excision and careful mapping of the peripheral and deep margins of horizontal frozen sections allow for comprehensive examination of all the borders of the excised tissue, resulting in excellent cure rates. In addition to the high cure rate, Mohs surgery is a tissue-sparing procedure that is an important advantage in cosmetically and functionally sensitive areas and contrasts with traditional approaches in which a set margin of excision is performed.

Indications for Mohs surgery include recurrent basal cell carcinomas (BCC) and squamous cell carcinomas (SCC), locations prone to recurrence (“H-zone” of the face: inner canthus, nasolabial fold, nose, periorbital, temple, upper lip and periauricular regions, retroauricular, and chin), at/near critical structures (e.g., eye, lip), large tumors (>2 cm), ill-defined tumor margins, aggressive histology (BCC - morpheaform infiltrative, basosquamous, perineural; SCC - poorly differentiated, invasive, perineural), and special hosts (immunosuppressed, basal cell nevus syndrome, xeroderma pigmentosum). Therefore, in this patient, the primary indication for Mohs surgery would be the recurrent nature of her BCC.

2017

How well did you know this?
1
Not at all
2
3
4
5
Perfectly
15
Q

A 25-year-old obese man is evaluated because of new onset of inflamed nodules involving the bilateral axillae. A diagnosis of hidradenitis suppurativa is made. Which of the following is the most appropriate initial medication for treatment of this patient’s condition?

A) Botulinum toxin type A
B) Clindamycin
C) Cyclosporine
D) Etanercept
E) Prednisone

A

The correct response is Option B.

Hidradenitis suppurativa is a chronic inflammatory skin disease. Also known as acne inversa, it is characterized by recurrent nodules and abscesses, typically of apocrine gland–bearing skin.

This patient has mild hidradenitis, with an initial presentation of a few abscesses without sinus tracts or cicatrization/scarring.

Clindamycin applied topically is often used as a first-line therapy for mild hidradenitis. In a randomized, placebo-controlled trial, patients treated with twice-daily topical application of 1% clindamycin solution were found to have significantly less disease burden, and the treatment was well tolerated with few side effects.

Although there have been reports of the use of botulinum toxin in the treatment of hidradenitis, its role and efficacy in this setting are currently unclear.

Etanercept is a TNF-alpha inhibitor. Although some TNF-alpha inhibitors, particularly infliximab, have shown efficacy in patients with moderate-to-severe hidradenitis, data are conflicting with regard to Etanercept.

Prednisone is occasionally used to calm the inflammatory process in severe hidradenitis. However, it does not prevent formation of new lesions and is rarely used for long-term therapy in patients with hidradenitis because of possible adverse effects.

A few case reports have described improvement with cyclosporine in refractory cases of hidradenitis. However, it is typically not used for initial medical treatment of hidradenitis, and duration of use is often limited by adverse effects.

2017

How well did you know this?
1
Not at all
2
3
4
5
Perfectly
16
Q

A 75-year-old woman is evaluated because of a new skin lesion on the right upper eyelid. Examination of the specimen obtained on biopsy shows a 1-cm Merkel cell carcinoma. In addition to regional node sampling, which of the following is the most appropriate excision and adjuvant management in this patient?

A) 1-cm margins and chemotherapy
B) 2-cm margins and chemotherapy
C) 1-cm margins and postoperative radiation therapy
D) 2-cm margins and postoperative radiation therapy
E) 5-mm margins and postoperative radiation therapy

A

The correct response is Option C.

Merkel cell carcinoma, an aggressive neuroendocrine tumor, is most likely. It presents in older, immunocompromised women in sun-exposed areas. About 80% of Merkel cell carcinomas are secondary to polyomavirus infection. Treatment of the primary tumor should be wide local excision or Mohs micrographic surgery. For wide local excision of tumors smaller than 2 cm, the recommended surgical margin should be 1 cm. As there is a high rate of occult nodal metastasis, and nodal status is associated with mortality rates, biopsy of the sentinel node is recommended for all cases regardless of primary tumor size. Merkel cell carcinoma is a radiosensitive tumor, and postoperative adjuvant radiation therapy has been shown to decrease local recurrence. Chemotherapy is only currently indicated for palliation and distant metastasis.

2017

How well did you know this?
1
Not at all
2
3
4
5
Perfectly
17
Q

A 62-year-old man is evaluated because of a new skin lesion in his back. Excision of the lesion with administration of a local anesthetic agent is planned. Medical history shows hypertension and coronary artery disease treated with coronary balloon angioplasty 10 years ago. He takes 81 mg of aspirin daily. Preoperatively, which of the following is the most appropriate aspirin regimen for this patient?

A) Maintaining current dosage
B) Withholding for 1 day
C) Withholding for 2 days
D) Withholding for 7 days
E) Withholding for 14 days

A

The correct response is Option A.

For this patient with increased risk of cardiovascular events, the safest approach is not to withhold aspirin prior to dermatologic surgery.

Aspirin inhibits platelet aggregation by irreversibly binding to the cell’s cyclooxygenase enzyme stores, blocking the production of thromboxane. Its use at lower doses in long-term primary and secondary prevention of cerebrovascular and cardiovascular thrombotic events is well established.

Withholding of daily aspirin in patients with known cardiovascular disease can lead to a platelet rebound phenomenon featuring increased thromboxane production, decreased fibrinolysis, and a prothrombotic state. Discontinuation of oral antiaggregants has been found to be an independent predictor of both death and major ischemic events.

Several studies revealed no significant increase in the risk of bleeding complications after cutaneous surgery in patients who continued taking aspirin perioperatively.

2017

How well did you know this?
1
Not at all
2
3
4
5
Perfectly
18
Q

A 20-year-old man comes to the office for definitive treatment of a punch biopsy–proven dermatofibrosarcoma protuberans (DFSP) of the scalp. Which of the following is the most appropriate treatment plan?

A) Cryoablation
B) Radiation
C) Referral for Mohs micrographic surgery
D) Resection with 0.5-cm margin
E) Resection with 1-cm margin

A

The correct response is Option C.

Dermatofibrosarcoma protuberans (DFSP) is relatively uncommon, soft-tissue-only tumor that is locally aggressive. Since it can extend along connective tissues along the deep layers, margins required for DFSP need to be no less than 2 cm for an acceptable non-recurrence rate, and even then it is quite high. Predicted rate of recurrence is anywhere from 11 to 20% with a 3-cm margin.

Mohs micrographic surgery has demonstrated to have a much lower recurrence rate, with multiple studies demonstrating less than 10%. Subsequently, Mohs micrographic surgery is the best initial treatment plan for complete resection of DFSP.

Radiation therapy is contraindicated for DFSP. Cryoablation is employed for precancerous skin lesions.

2017

How well did you know this?
1
Not at all
2
3
4
5
Perfectly
19
Q

Which of the following patients’ skin lesions is most suggestive of malignancy on the basis of its clinical features?

A) A 14-year-old boy with a 5-mm, round, brown macule present since birth on the distal aspect of the thigh
B) A 22-year-old woman with a 6-mm friable, pedunculated papule present for 6 weeks on the cheek
C) A 55-year-old man with a 5-mm pink, pearly papule present for 6 months on the lateral nasal sidewall
D) A 70-year-old man with a 1.5-cm waxy, yellow and brown, scaly plaque present for several years on the left temple

A

The correct response is Option C.

Once an individual’s personal and family histories are considered, the history and appearance of a suspicious skin lesion will provide important information and provide clues about a diagnosis.

The “ABCD” (Asymmetry, Border irregularity, Color variegation, Diameter >6 mm) criteria are important for assessing potential malignancy of pigmented lesions. A flat, unchanging pigmented lesion in a young patient is unlikely to be malignant.

A pyogenic granuloma classically presents after local trauma and is characterized by a friable papule that bleeds easily.

Basal cell carcinoma and squamous cell carcinoma are the most common cutaneous skin malignancies. They may present as discrete, slowly growing lesions that have a history of bleeding and ulceration. Basal cell malignancies have a characteristic appearance of round, oval nodules with a shiny, pearly appearance and overlying telangiectasias.

Seborrheic keratoses are common benign skin lesions found in old age. They are a proliferation of immature keratinocytes and have a characteristic appearance of being well circumscribed, scaly, and have a “stuck on” appearance. While unsightly, these lesions have no malignant potential.

2017

How well did you know this?
1
Not at all
2
3
4
5
Perfectly
20
Q

A 62-year-old woman with biopsy-proven basal cell carcinoma of left mid cheek presents for consultation for surgical treatment. On physical examination, the lesion is 0.6 cm in diameter and has indistinct borders. Which of the following criteria is the most likely indication for Mohs micrographic surgery in this patient?

A) Anatomical location of the cancer
B) Diagnosis of basal cell carcinoma
C) Indistinct borders
D) Patient age
E) Size of the carcinoma

A

The correct response is Option C.

In the case presented above, the strongest indication for Mohs micrographic surgery comes from the anatomic finding of indistinct borders. Other findings are not necessarily indications for Mohs. Other proven indications are recurrent cancer, high risk zones of the face, morpheaform basal cell cancer, or evidence of neurovascular involvement. By some studies, larger sized basal cell cancers (>2 cm) may be indications.

By definition, Mohs micrographic surgery is a technique for treatment of complex or ill-defined skin cancers with examination of 100% of tissue. One physician performs the procedure, acting in two distinct capacities: surgeon (excising the cancer) and pathologist (reading the slides). Usually, the final pathologic clearance is given on the same day as the resection. This is in comparison with a wide local excision technique, which is performed by two different physicians: a surgeon who removes the cancer and a pathologist who reads it separately. In the latter scenario, although a “wet” read can be done on the same day, the final pathologic evaluation has to await the permanent preparation of slides and a final read. Also, using the routine pathologic evaluation, only 2% of the margins are looked at (using the common “bread loafing” technique), compared with the 100% of tissues evaluated using the Mohs technique.

The vast majority of Mohs resections is done with one or two excisions. Although at times there is a need for multiple excisions, the need for excisions is driven by the positive margins detected pathologically. In Mohs technique, the amount of healthy tissue taken to obtain “clear” margins is the minimum needed to do so, thus preserving the healthy tissue, which can be critically useful in the ensuing reconstruction. These margins are definitively assessed on that same day of surgery, so once the patient leaves the Mohs surgeon’s office, he or she knows the cancer has been completely removed. This is why the margin control is superior in Mohs technique compared with other surgical techniques.

2016

How well did you know this?
1
Not at all
2
3
4
5
Perfectly
21
Q

A 75-year-old man comes to the office because of a 5-year history of a pruritic lesion on the right groin that has been enlarging gradually in size. A photograph is shown. Examination of a specimen obtained on biopsy shows Paget disease. Which of the following is the most appropriate next step in management?

A) Oral miltefosine
B) Topical hydrocortisone
C) Topical miconazole
D) Wide excision
E) Observation

A

The correct response is Option D.

This patient described has extramammary Paget disease and the treatment is wide excision. Paget disease of the breast also presents with eczematous skin changes and is associated with breast cancer. Extramammary Paget disease, however, is an intraepithelial carcinoma that commonly involves the vulvar, perianal, perineal, scrotal, and penile regions. It presents as well-defined, moist, erythematous plaques associated with pruritis. Histopathologic examination shows epidermal acanthosis and elongated rete ridges. Paget cells are large intraepidermal cells with a large nucleus and abundant pale cytoplasm. There is a 7 to 40% rate of associated malignancy. Wide excision is the standard of care, and recent reports have shown that Mohs micrographic surgery can improve evaluation of resection margins.

Observation is not appropriate for extramammary Paget disease given the potential risk for malignancy. Topical treatment with steroids or antifungals is also not appropriate. Oral miltefosine is a treatment for leishmaniasis, an infectious disease that can involve the skin, mucous membranes, and internal organs. Although cutaneous leishmaniasis can present with ulcerating lesions or a dense dermal infiltrate, the histology is predominantly histiocytes, lymphocytes, and plasma cells.

2016

How well did you know this?
1
Not at all
2
3
4
5
Perfectly
22
Q

An otherwise healthy 65-year-old man comes to the clinic because of a 3-cm ulcerated lesion of the scalp. There are no palpable regional lymph nodes. Chest x-ray study shows no abnormalities. A punch biopsy is performed and a diagnosis of well-differentiated squamous cell carcinoma is made. Wide local excision of the lesion is planned. Which of the following is the recommended minimum surgical margin in this patient?

A) 1 mm
B) 2 mm
C) 4 mm
D) 6 mm
E) 12 mm

A

The correct response is Option D.

The most appropriate surgical margin recommended for the clinical scenario described is 6 to 10 mm.

Cutaneous squamous cell carcinoma (cSCC) is the second most common skin cancer after basal cell carcinomas. They are broadly categorized into low- and high-risk lesions depending on size, location, depth of invasion, recurrence, and patient factors such as immunosuppression. A 4-mm margin of healthy tissue is recommended for lower-risk lesions. This category includes well-differentiated tumors smaller than 2 cm in diameter that do not occur on the scalp, ears, eyelids, lips, or nose, and do not involve subcutaneous fat. Therefore, simple excision is most valuable in the treatment of small primary squamous cell carcinomas on the trunk, extremities, or neck, where tissue sparing is less essential. The recurrence rate after the excision of low-risk lesions ranges from 5 to 8%.

A 6-mm margin of healthy tissue is recommended for lesions that are larger than 2 cm, invasive to fat, or in high-risk locations (i.e., central face, ears, scalp, genitalia, hands, feet). Given the cosmetic and functional impact of these wider margins, tumors in this latter category are often removed via Mohs micrographic surgery to achieve high cure rates while sparing healthy tissue. The depth of an excision should always include a portion of the subcutaneous fat.

Cure rates following simple excision of well-defined T1 lesions may be as high as 95 to 99%. The generally accepted 5-year cure rate for primary tumors treated with standard excision is 92%; this rate drops to 77% for recurrent cSCC. No large randomized studies have addressed the issue of appropriate margin size in cSCC, as has been done for melanoma. The recommendations for margin size should be taken only as rough guidelines, with the understanding that large, aggressive lesions frequently have substantial extension beyond the apparent superficial boundary. Therefore, a surgeon’s experience and judgment when planning surgical margins is paramount to the successful treatment of cSCC.

2015

How well did you know this?
1
Not at all
2
3
4
5
Perfectly
23
Q

A 24-year-old woman is evaluated because of a slow-growing subcutaneous mass of the scalp. An excisional biopsy is performed, and pathologic examination shows keratin and its breakdown products. Which of the following is the most likely origin of this lesion?

A) Adipocyte
B) Capillary
C) Hair follicle
D) Mechanoreceptor
E) Sebaceous gland

A

The correct response is Option C.

Pilar cysts, also known as trichilemmal cysts, originate from the outer root sheath of the hair shaft. They present as firm, slow-growing subcutaneous nodules, and may be difficult to differentiate clinically from epidermoid cysts. They are commonly found on the scalp where they are the most common cutaneous cyst. They are lined by stratified squamous epithelium, which undergoes keratinization. In some cases, these lesions can demonstrate aggressive biologic behavior (proliferating trichilemmal tumors, malignant proliferating trichilemmal tumors), in which case they should be completely excised.

Lesions that originate from adipocytes include lipomas and angiolipomas.

Lesions of vascular origin include cherry angiomas and pyogenic granulomas.

2015

How well did you know this?
1
Not at all
2
3
4
5
Perfectly
24
Q

A 45-year-old woman is evaluated because of a 1-year history of skin abnormalities. Physical examination shows areas of thickened skin involving the forearms and hands. Telangiectasias are seen on the face and oral cavity. A review of systems discloses symptoms of heartburn and dysphagia. Which of the following is the most likely diagnosis?

A) Amyloidosis
B) Dermatomyositis
C) Hypothyroidism
D) Scleroderma
E) Systemic lupus erythematosus

A

The correct response is Option D.

The scleroderma spectrum of disorders includes localized scleroderma and systemic sclerosis, both of which are characterized by thickened sclerotic skin. Localized scleroderma involves only the skin, whereas systemic sclerosis is associated with extracutaneous involvement and is subcategorized into limited and diffuse forms. Limited cutaneous systemic sclerosis is restricted to the hands, distal forearm, face, and neck. Limited scleroderma is sometimes referred to as CREST syndrome, which is an acronym for the following features: calcinosis, Raynaud syndrome, esophageal dysmotility, sclerodactyly, and telangiectasia. Diffuse cutaneous systemic sclerosis includes truncal involvement (chest, abdomen, upper arms, shoulders).

In most patients with systemic sclerosis, there is gastrointestinal involvement. Symptoms are present in more than half of patients and most commonly are related to gastroesophageal reflux disease, resulting in symptoms such as heartburn and dysphagia. Other findings in systemic sclerosis include: diarrhea, mucocutaneous telangiectasia of the face/lips/oral cavity/hands, dyspnea on exertion/interstitial pulmonary disease, and digital infarctions/pitting.

Amyloid infiltration of the skin may produce thickening and stiffness. Telangiectasias are not a feature of amyloidosis. Gastrointestinal involvement with amyloid typically does not cause esophageal reflux or heartburn.

Dermatomyositis is an idiopathic inflammatory myopathy characterized by skin and muscle abnormalities. Cutaneous involvement manifests in the form of distinct rashes, such as of the upper eyelids (heliotrope rash), hands (Gottron sign), and chest and shoulders (shawl sign). However, thickened skin of the trunk and limbs, and telangiectasias are not features of dermatomyositis.

Hypothyroidism can result in cutaneous and dermal edema (myxedema). Other findings include fatigue, cold intolerance, weight gain, constipation, dry skin, myalgia, and menstrual irregularities. It is not associated with telangiectasias.

Systemic lupus erythematosus (SLE) is a chronic inflammatory disease that can affect the skin, joints, kidneys, lungs, nervous system, serous membranes, and/or other organs of the body. Mucocutaneous changes include butterfly rash, erythematous plaques (discoid lesions), and ulcers. SLE is not associated with telangiectasias.

2015

How well did you know this?
1
Not at all
2
3
4
5
Perfectly
25
Q

A 55-year-old woman comes to the office because of a 3-year history of a raised, dark brown lesion on her back. The lesion is not painful and does not bleed. A photograph is shown. An excisional biopsy is performed. Pathologic examination shows proliferation of cells from the basal layer of the epidermis with cystic inclusions. The lesion exhibits hyperkeratosis, acanthosis, and papillomatosis. Which of the following is the most likely diagnosis?

A) Basal cell carcinoma
B) Malignant melanoma
C) Nevus sebaceous
D) Seborrheic keratosis
E) Verrucous carcinoma

A

The correct response is Option D.

The lesion is a seborrheic keratosis, also known as verruca senilis or pigmented papilloma. Seborrheic keratoses are common benign lesions that may begin to appear during the fifth decade of life. They arise from the basal layer of the epidermis and are composed of well-differentiated basal cells. They often contain cystic “inclusions” of keratinous material called “horn cysts.” Lesions exhibit hyperkeratosis (thickening of the stratum corneum), acanthosis (diffuse epidermal hyperplasia and thickening of the skin), and papillomatosis (skin surface elevation). The growth and depth of pigmentation vary directly with exposure to sunlight. If left untreated, they will enlarge gradually and increase in thickness. Seborrheic keratoses typically do not involute spontaneously. They may appear on the head, neck, and trunk after age 50 years and are often distinctly marked and have a waxy, stuck-on appearance. The surface is soft and oily to the touch. Sizes can range from 1 mm to 5 cm. No treatment is necessary, but these lesions are cosmetically unappealing, and for that reason alone, patients may request to have them removed. It is extremely rare for cutaneous malignancies to develop within seborrheic keratoses.

A verrucous carcinoma is a variant of squamous cell carcinoma. As such, it requires wide local excision with negative margins for treatment.

Nevus sebaceous (Jadassohn nevus) may be described as cerebriform, nodular, or verrucous. It is hairless and can appear on the scalp, face, or neck. When present at birth, it persists throughout life and tends to become more verrucous and nodular during the growth phase associated with puberty. Over time, these lesions are associated with a risk of basal cell carcinoma, which occurs in approximately 15 to 20%. Keratoacanthoma and squamous cell carcinoma may also develop, although with much less frequency than basal cell carcinoma. Because of the risk of malignant transformation, complete excision is generally recommended.

Melanoma results from malignant transformation of the melanocyte; the pigment-producing cell of the body can occur anywhere melanocytes are present, including skin, eye, and the mucous membranes of the upper digestive tract, sinuses, anus, and vagina. The incidence of cutaneous melanoma in the United States has increased steadily over the past 50 years and is now 15 per 100,000. Worldwide, the incidence of melanoma is generally reflective of variation in genetic, phenotypic, and ultraviolet (UV) exposure risk factors. Major risk factors include exposure to UV radiation and genetic predisposition. The exposure risk primarily involves intermittent, damaging exposure to the sun such that history of a severe sunburn (blistering or pain for more than 2 days), even in youth, confers an approximately twofold increase in risk. Patients who have fair skin, blue eyes, red hair, and are prone to freckling are at increased risk for melanoma. Mutations in two genes are associated with hereditary melanoma predisposition.

2015

How well did you know this?
1
Not at all
2
3
4
5
Perfectly
26
Q

A 60-year-old woman comes to the office for evaluation of a firm, violaceous, 3-cm lesion of the left cheek. An incisional biopsy shows a Merkel cell tumor. The residual margins are positive. Which of the following is the most appropriate management?

A) Chemotherapy
B) Laser ablation
C) Mohs micrographic surgery
D) Radiation therapy and chemotherapy
E) Wide local excision and radiation therapy

A

The correct response is Option E.

Merkel cell tumor is an unusual and highly aggressive skin cancer. More than 50% of Merkel cell tumors occur in the head and neck region. Risk factors for Merkel cell tumors are exposure to sun and ultraviolet light, and immunosuppression. There is a recent association with Merkel cell polyomavirus.

Merkel cell generally presents as a firm, painless nodule (up to 2 cm in diameter) or as a mass (>2 cm in diameter). Although classically described as red in color, it may be flesh-colored or blue. It often enlarges rapidly.

The standard of management is surgical excision combined with radiation therapy. Radiation therapy decreases local recurrence rates. Node-negative patients with no distant metastasis treated with surgery and radiation have 5-year survival rates of approximately 90%. Mohs micrographic surgery and wide local excision are both accepted modalities of surgical resection. It is well known that surgery alone is insufficient to cure or control Merkel cell tumors. Consideration should be given to evaluation of the lymph nodes. Sentinel node biopsy is a common modality.

2015

How well did you know this?
1
Not at all
2
3
4
5
Perfectly
27
Q

A 14-year-old girl with Fitzpatrick Type V skin comes to the office for evaluation of a nevus on the right side of the face. Physical examination shows a macular, bluish grey, irregular area of hyperpigmentation involving the right infrapalpebral region, nasolabial fold, and zygomatic region. Pigmentation of the right sclera is noted. Which of the following is the most appropriate treatment for this lesion?

A) Camouflage therapy
B) Cryotherapy
C) Dermabrasion
D) Mohs micrographic excision
E) Q-switched ruby laser

A

The correct response is Option E.

This patient has the acquired form of nevus of Ota, also known as nevus fuscoceruleus ophthalmomaxillaris or oculodermal melanocytosis, a dermal melanocytic hamartoma that demonstrates bluish hyperpigmentation along the ophthalmic and maxillary divisions of the trigeminal nerve. The failure of complete embryonic migration of melanocytes from the neural crest to the epidermis results in dermal nesting with the resultant dermal melanin causing the Tyndall effect. This disorder primarily affects darker-pigmented individuals and is more prevalent in females. It has a bimodal age incidence, with a peak at 1 year of age and a second around puberty. The lesion tends to become increasingly prominent with age, puberty, and postmenopausal state. Most patients have no family history. Ophthalmologic examination is recommended because of a reported 10% association with ipsilateral glaucoma. Malignant degeneration to melanoma occurs in approximately 4% of reported cases and is more frequent in lighter-skinned individuals. Diagnosis is mainly clinical with confirmatory biopsy indicated when the diagnosis is uncertain or in rapidly expanding or nodular lesions suggestive of malignancy.

The most effective treatment option is laser therapy, particularly with a Q-switched laser with ruby (694 nm), alexandrite (755 nm), or neodymium: yttrium-aluminum-garnet (1064 nm). The wavelength, pulse duration, and energy densities inherent in the Q-switched laser provide the desired parameters for melanin photothermolysis. Dyspigmentation is a possible complication, although it is mostly transient.

Before the advent of laser therapy, treatment options were suboptimal. Makeup or camouflage therapy offered only temporary improvement. Dermabrasion followed by cryotherapy had the potential for dermal scarring and atrophy and was ineffective for those lesions with deep dermal melanocytes. Surgical excision options were also associated with scarring. Mohs micrographic excision has not been described for excision of these lesions.

2014

28
Q

An 87-year-old Caucasian man comes to the office with multiple 5- to 6-mm lesions on the forehead. He has a long history of sun exposure. Physical examination shows the lesions are erythematous, rough, and scaly. Which of the following is the most appropriate treatment?

A) Dexamethasone
B) Docosanol
C) Imiquimod
D) Isotretinoin
E) Observation with 1-month follow up

A

The correct response is Option C.

This patient’s lesions are most consistent with actinic keratoses. Actinic keratoses are most commonly seen in fair-skinned individuals in areas that have had long-term sun exposure. They are the most common skin lesions to demonstrate malignant potential and may progress to squamous cell carcinoma.

Given the propensity of actinic keratoses to malignant transformation, treatment is generally recommended over observation. For multiple lesions, topical agents are generally effective and well tolerated. Imiquimod is thought to exert its effects by inducing a local immune response as well as apoptotic pathways. Other effective treatments include photodynamic therapy, cryotherapy, 5-fluorouracil, and diclofenac gel.

Dexamethasone is a corticosteroid typically used for inflammatory or autoimmune skin conditions. Isotretinoin is used to treat cystic acne. Docosanol is an antiviral medication used for herpes simplex.

2014

29
Q

A 55-year-old woman is evaluated for a biopsy-proven squamous cell carcinoma of the right preauricular area measuring 2.1 cm in diameter. She is otherwise healthy. Which of the following is the most appropriate next step in management?

A) Electrodessication of the lesion
B) Excision of the lesion with frozen sections
C) Excision with a 2-mm margin
D) Excision with a 4-mm margin
E) Topical application of 5% fluorouracil

A

The correct response is Option B.

Successful local treatment of squamous cell carcinoma of the skin depends significantly on whether the tumors are at high or low risk for the complications of recurrence and metastasis. The external ear, lips, nose, and scalp appear to be high-risk locations for squamous cell carcinoma of the skin.

Squamous cell carcinomas of the skin larger than 2 cm are twice as likely to recur locally and three times as likely to metastasize than tumors that are less than 2 cm in diameter.

Frozen intraoperative examination of specimen edges can be used to judge thoroughness of excision before closure. Frozen sections of margins are recommended for high-risk squamous cell carcinoma and basal cell carcinoma in high-risk areas, lesions more than 2 cm, and any morpheaform basal cell carcinoma.

Electrodessication has excellent cure rates in small, low-risk squamous cell carcinoma of the skin.

Topical application of 5% fluorouracil has a role in the treatment of diffuse actinic keratoses of the face.

Surgical excision is subdivided into excision with standard margins, excision with frozen-section margin evaluation, and Mohs micrographic surgery. For low-risk non-melanoma skin cancers extending into the dermis only, excision with standard margins (4 mm for basal cell carcinoma) is the usual treatment. Adequate margins of 4 mm for low-risk squamous cell carcinoma and 6 mm for high-risk squamous cell carcinoma have been demonstrated by direct tumor extension from the clinical margin but are not necessarily an estimate of cure rate.

Squamous cell carcinomas are slower to invade deeper tissue than are cutaneous malignant melanomas.

2014

30
Q

A 55-year-old woman with a BMI of 32 kg/m2 comes to the office with advanced hidradenitis suppurativa of the groin, lower abdomen, and upper thigh. Which of the following treatments is most likely to have the greatest likelihood of success in this patient?

A) Antibiotics and excision
B) Antibiotics and percutaneous drainage
C) Clindamycin irrigation
D) Intralesional injection of a corticosteroid
E) Sclerotherapy

A

The correct response is Option A.

Percutaneous drainage of hidradenitis suppurativa pustule and fistula tracts, although a plausible short-term fix to address the immediate symptoms, does little to ensure long-term resolution of this very difficult clinical entity with marked impact on quality of life. Addition of topical antibiotic washes or oral antibiotics to percutaneous drainage had no significant effect on long-term recurrence rates.

Hidradenitis suppurativa is a recurrent inflammatory disease of the apocrine glands. It initially develops from follicular occlusion with subsequent abscess, inflammation, fistulas, sinus tracts, and scarring. The sites most commonly affected are the intertrigonal regions such as the axilla, groin, and genital/anal region; although, it can also affect the breasts, hips, and thighs. Women are affected three times as often as men.

Initial treatment involves local wound care and antibiotic therapy. For advanced disease, this may be followed by excision of the area of high-density apocrine glands with minimal undermining and direct closure at the site of the hidradenitis wound. At the site of inadequate resection of an area of infected glands, or if there is a recurrence, radical resection yields the best long-term result. Skin grafting and fasciocutaneous and musculocutaneous flaps have been described to cover the excisional defect. The musculocutaneous flap has been reported to be a valid option for managing infected lesions because of the abundant blood supply. Delayed secondary wound closure, with or without vacuum-assisted closure or skin substitutes, has also shown plausible outcomes.

Sclerotherapy has no role in treatment of hidradenitis suppurativa.

2014

31
Q

An otherwise healthy 30-year-old woman is diagnosed with dermatofibrosarcoma protuberans (DFSP) of the upper back. A wide excision is performed, and a local flap is used to reconstruct her back. On follow-up evaluation, CT scan shows multiple pulmonary metastases. Which of the following is the most appropriate next step in management?

A) Chemotherapy
B) Hormone therapy
C) Immunotherapy
D) Radiation therapy
E) Surgical excision

A

The correct response is Option A.

The most appropriate next step in management is chemotherapy. Patients with inoperable, recurrent, or metastatic disease may benefit from imatinib which is a tyrosine kinase inhibitor and acts as a molecularly targeted drug. It acts by inhibiting the platelet-derived growth factor receptor tyrosine kinase. Dermatofibrosarcoma protuberans (DFSP) is characterized by chromosomal rearrangements resulting in the production of platelet-derived growth factor B, eventually leading to autocrine growth stimulation of DFSP. Imatinib functions as an inhibitor of platelet-derived growth factor receptors, thus blocking this autocrine stimulation. Therefore, imatinib can be used as an adjuvant therapy for cases in which obtaining sufficient surgical margins is impossible. Neoadjuvant imatinib has also been used for locally advanced primary tumors.

Radiation therapy may improve local control and reduce the risk of recurrence postoperatively in patients with DFSP. There is no described role of it in pulmonary metastases. Resection of the multiple lesions in the lung, and hormonal or immune therapy are not recommended for metastatic DFSP.

2019

32
Q

An otherwise healthy 20-year-old woman develops a keloid on her right earlobe after an ear piercing. Excision and radiation therapy are planned. Which of the following is the ideal time after excision for the initiation of radiation therapy?

A) 1 day
B) 7 days
C) 2 weeks
D) 4 weeks
E) 6 weeks

A

The correct response is Option A.

The ideal time to give radiation therapy in this case is on postoperative day one. Earlobes and the helix of the auricle are common sites for keloid formation, usually after trauma or ear piercing, with an incidence of approximately 2.5%. There are numerous adjuvant therapies (eg, radiation therapy), medical therapies, (eg, intralesional steroids, 5-fluorouracil interferon, and topical silicone), and physical approaches (eg, pressure) that can be used in addition to excision of the keloid to help reduce its recurrence. However, the treatment options for such lesions are still controversial, because there are numerous challenges, and no single best treatment or best combination of treatments has been proved to manage these conditions effectively.

Radiation therapy has long been known to be effective in the early phase of wound healing. More specifically, radiation therapy is sensitive to endothelial vascular buds and decreases proliferation of new fibroblasts. As such, radiation therapy after keloid excision should be performed as soon as possible. Usually, better results are reported when radiation therapy is performed within 1 to 3 days after surgery. The most commonly administered doses are between 10 and 15 Gy over a period of 2 or 3 days.

The other options are incorrect as they are outside the ideal time frame.

2019

33
Q

A 65-year-old man comes to the office with a 2-cm basal cell carcinoma involving the left nasal ala extending across the nasofacial junction onto the cheek. A photograph is shown. Medical history includes previous melanoma. Which of the following is an indication for Mohs micrographic surgery over conventional excision in this patient?

A) Cancer size
B) Diagnosis of basal cell carcinoma
C) History of previous melanoma
D) Location of the lesion
E) Patient age

A

The correct response is Option D.

Mohs micrographic surgical technique has demonstrated cure rates of 99% for primary basal cell carcinomas and up to 95% for recurrent basal cell carcinomas. In this particular patient, the strongest indication for use of the Mohs technique is the anatomical location. The nose is considered a high-risk location in the classically described “H-zone.” This patient underwent Mohs excision with multi-stage forehead flap reconstruction, as shown in the photographs. Patient age, history of previous melanoma, and tumor size 2 cm or less are not standard indications for Mohs excision. Other indications for Mohs technique include the following: Recurrent basal cell/squamous cell carcinomas, Locations prone to recurrence- “H-zone” of the face: periorbital, periauricular, temple, upper lip, nose/nasolabial fold, and chin, Tumors involving critical structures such as the eyelid or lip, Functionally important areas such as the genitals, perianal location, hands, and feet, Tumors arising in sites of previous irradiation therapy, Large tumors (> 2 cm) Lesions with ill-defined tumor margins, Histologic aggressive subtype (morpheaform, basosquamous, perineural, and invasive/poorly differentiated squamous cell carcinoma) Tumors arising in immunosuppressed patients such as transplant recipients or patients with genetic predisposition (basal cell nevus syndrome, xeroderma pigmentosum)

2019

34
Q

A 76-year-old man comes to the office for follow-up after undergoing Mohs resection of a basal cell carcinoma (BCC) of the left central cheek. He smokes a half pack of cigarettes daily and has no intention of quitting. Final margins have been confirmed to be free of residual tumor. The defect is full-thickness skin and measures 2 × 2 cm. Which of the following is the most appropriate treatment option for this patient?

A) Cervicofacial flap
B) Direct primary closure
C) Full-thickness skin grafting
D) Healing by secondary intention
E) Rhomboid transposition flap

A

The correct response is Option B.

Direct primary closure is the best option for this patient. The cheek skin has ample laxity and allows for closure of small- to medium-sized cheek defects. In general, closure should be aligned with the relaxed skin tension lines, and aggressive management of dog ears should be performed. Direct linear closure also allows for simple monitoring for recurrence without any distortion of the anatomy. Secondary intention is better utilized in areas of concavity, like the medial canthus. Full-thickness skin grafting, rhomboid flaps and cervicofacial flaps will have a higher complication rate in an active smoker, while likely providing a lesser aesthetic outcome given the poor orientation of scars.

2019

35
Q

A 68-year-old woman presents with a 3-cm morpheaform basal cell carcinoma (BCCA) involving the left mid cheek. A photograph is shown. Which of the following is an indication for Mohs micrographic surgery over conventional excision?

A) Anatomic location of lesion
B) Diagnosis of BCCA
C) History of previous melanoma
D) Morpheaform subtype
E) Patient age

A

The correct response is Option D.

Mohs micrographic surgical technique has demonstrated cure rates of 99% for primary basal cell carcinomas (BCCA) and up to 95% for recurrent BCCAs. In this particular patient, the strongest indication for utilization of the Mohs technique is the more aggressive morpheaform subtype of BCCA. This patient underwent Mohs excision with adjacent tissue transfer reconstruction. Patient age, history of previous melanoma, and the diagnosis of BCCA (without aggressive features) are not indications for Mohs excision.

Other indications for Mohs technique include the following:

Recurrent basal cell/squamous cell carcinomas;

Locations prone to recurrence—“H-zone” of the face: periorbital, periauricular, temple, upper lip, nose/nasolabial fold, and chin;

Tumors involving critical structures such as the eyelid or lip;

Functionally important areas such as the genitals, perianal location, hands, and feet;

Tumors arising in sites of previous irradiation therapy;

Large tumors (greater than 2 cm);

Lesions with ill-defined tumor margins;

Histologic aggressive subtype (morpheaform, basosquamous, perineural, and invasive/poorly-differentiated squamous cell carcinoma);

Tumors arising in immunosuppressed patients, such as transplant recipients or patients with genetic predisposition (e.g. basal cell nevus syndrome, xeroderma pigmentosum).

2020

36
Q

A 6-month-old female infant is referred by the pediatrician for management of a skin lesion on the right parietal scalp that was noticed at birth. The lesion is a 2 × 1-cm yellow plaque that is devoid of hair and has grown in proportion with the child. She is otherwise healthy and is doing well. Which of the following is the most appropriate recommendation for the child’s parents?

A) Biopsy to rule out malignancy
B) CO2 laser therapy prior to puberty
C) Excision due to high risk of malignant transformation
D) MRI to evaluate for brain abnormalities
E) Continued observation because of anesthetic risk

A

The correct response is Option E.

These clinical features are typical of nevus sebaceous. They present as yellow-orange flat plaques, occurring most commonly on the scalp (60%) or face (30%). They are usually present at birth but may appear in the first few years of life. They are hamartomas, arising from the pilosebaceous units of the skin. They occur due to mutations in the RAS pathway. Maternal transmission of genetic material from the human papilloma virus to the fetus has been implicated as a causative factor.

Excision of nevus sebaceous is performed because of the cosmetic concerns and risk of secondary tumors. The most common neoplasia is trichoblastoma, which is a benign tumor, although more than 40 types of secondary tumors have been described. The most common malignant tumor is basal cell carcinoma. Initial studies reported the risk of malignant transformation to be 10%, however, more recent studies indicate that this number is 1%. The risk of malignant transformation increases with age; it is extremely rare in childhood and has not been reported in children younger than 5 years of age. The risk of malignant transformation is, thus, very small and in the absence of any morphologic change in the lesion, biopsy is not indicated.

Nevus sebaceous lesions undergo change in appearance during puberty and become thick and verrucous, presumably due to hormonal influence. Most practitioners thus recommend definitive treatment prior to puberty. Surface ablative therapies like electrodessication, curettage, dermabrasion, photodynamic and CO2 laser have been proposed to improve the appearance of these lesions. However, nevus cells can be left behind in the deeper layers, with the risk of developing secondary tumors and potentially making future detection of neoplastic change more difficult.

The definitive treatment of nevus sebaceous is full thickness skin excision. In December of 2016, the Food and Drug Administration (FDA) issued a warning that “repeated or lengthy use of general anesthetic and sedation drugs during surgeries or procedures in children younger than three years or in pregnant women during their third trimester may affect the development of children’s brains.” The FDA modified the warning in April of 2017, stating “consideration should be given to delaying potentially elective surgery in young children where medically appropriate.”

Most of the data that lead to these warnings came from animal studies that showed learning and behavioral problems after exposure to anesthetics that block N-methyl-D-aspartate (NMDA) and gamma-aminobutyric acid (GABA). Research in humans is not conclusive, with some studies indicating neurotoxicity with multiple exposures, but not with a single exposure. However, the duration of a “brief exposure” has not been well-defined. There are ongoing studies that will hopefully shed further light on the matter. In view of this, it may be prudent to delay elective procedures in children if this will be not detrimental to the child’s health or final outcome.

Numerous syndromes are associated with nevus sebaceous. These mostly involve the central nervous and ocular system, but can also involve other organs. There does not appear to be a correlation between size of skin lesions and risk of nervous system involvement, but large lesions and centrofacial location have been suggested as having higher risk. Small isolated nevus sebaceous lesions in the absence of any other systemic manifestations do not warrant central nervous system imaging or systemic work up. The vast majority of nevus sebaceous are isolated lesions.

2020

37
Q

A 32-year-old Caucasian woman presents with multiple (>50) brown lesions on her arms and lower legs. They appear to be in areas of sun exposure. On examination, many of these lesions are well circumscribed, even in color, and less than 5 mm in size. The patient has a family history of melanoma. There are too many lesions to excise. Which of the following findings in one of these lesions would prompt an excisional biopsy?

A) Asymmetry
B) Clearly demarcated borders
C) Dark coloration
D) Waxy surface

A

The correct response is Option A.

In this patient with multiple melanocytic nevi, lesions should be treated with excisional biopsy if there is a high suspicion for melanoma. As there are more than 50 lesions, clearly there are too many to excise. These lesions should be evaluated for asymmetry, border irregularity, variable color, diameter greater than 6 mm, and evolution. Any of these signs in a lesion should lead to an excisional biopsy with a suspicion of melanoma, especially given the patient’s family history.

Lesions with a waxy surface are seborrheic keratoses and commonly found in an elderly population in sun-exposed areas. Dark coloration does not lead to a suspicion of melanoma.

2020

38
Q

A 38-year-old African American man presents with multiple purulent tunneling lesions in bilateral axillae and his right groin. Which of the following surgical treatments will result in the lowest likelihood of recurrence?

A) Deroofing
B) Electrosurgical peeling
C) Incision and drainage
D) Skin-tissue-saving excision
E) Wide excision

A

The correct response is Option E.

Hidradenitis suppurativa (HS) is an inflammatory skin disease with a characteristic clinical presentation of recurrent or chronic painful or suppurating lesions in the apocrine gland-bearing regions. HS should be differentiated from infections such as furuncles, carbuncles, and abscesses (due to infectious agents and response to antibiotics), cutaneous Crohn disease (often concurrent with gastrointestinal Crohn, which has “knife-cut” ulcers and no comedones [whiteheads or blackheads]), and acne (distributed on the face and upper truncus, whereas HS predominantly affects intertriginous areas).

Surgery is required to definitively treat the tunnels and scars associated with chronic HS. Although surgery is commonly recommended, the literature supporting surgical treatment is anecdotal, composed mostly of large case series or retrospective study reports. A systematic review by Mehdizadeh et al. concluded that a lower recurrence rate was found in procedures with wide excision (overall, 13%; primary closure, 15%; using flaps, 8%; grafting, 6%) compared with local excision (22%) or deroofing (27%). These operations can be disfiguring, and despite the removal of significant amounts of tissue, do not necessarily protect against disease recurrence.

2020

39
Q

A 74-year-old woman presents with biopsy-proven Merkel cell carcinoma involving the left cheek and no palpable lymphadenopathy. The most appropriate treatment involves wide local excision with 2-cm margins and which of the following?

A) Adjuvant radiation and immunotherapy
B) Adjuvant radiation only
C) Cervical lymphadenectomy
D) Sentinel lymph node biopsy and adjuvant radiation
E) No additional intervention is needed

A

The correct response is Option D.

Merkel cell carcinoma is a rare and aggressive neuroendocrine cutaneous malignancy with a propensity for lymph node and distant metastasis and high mortality. It typically presents as an asymptomatic erythematous papule or nodule in the head and neck region. Surgical treatment includes wide local excision of the primary lesion with 1- to 2-cm margins to achieve histologically clear margins. Sentinel lymph node biopsy has become standard of care and helps determine prognosis and may guide additional adjuvant therapy. Negative sentinel node biopsy correlates with longer disease-free survival, especially in early stage disease. Adjuvant radiation has been shown in improve local-regional control and increase survival. Cervical lymphadenectomy would only be indicated in the setting of clinically positive lymph nodes and is insufficient as the sole adjuvant therapy.

2020

40
Q

A 48-year-old man presents with an asymptomatic, slowly growing violaceous nodule overlying his right scapula. Dermoscopy shows a delicate pigmented network, vessels, and structureless light brown areas. Biopsy identifies the lesion as a dermatofibrosarcoma protuberans. To minimize recurrence, surgical excision should include which of the following?

A) Bone
B) 6-cm margin
C) Deep fascia
D) Frozen sections
E) Periosteum

A

The correct response is Option C.

Dermatofibrosarcoma protuberans (DFSP) is a rare neoplasm of intermediate malignancy. Taylor first described it in 1890, but Darier is credited with establishing DFSP as a distinct clinicopathological entity in 1924, and finally, Hoffman established the term in 1925.

Most DFSPs occur on the trunk (42%), followed by the upper extremities (23%), lower extremities (18%), then the head and neck (16%). The reason for recurrence is that microscopic projections are not removed adequately or assessed satisfactorily; the wider the margin, the higher the probability the tumor will be removed completely. Using 1-cm margins around the primary tumor leaves residual microscopic tumor in more than 70% of patients, 2-cm margins in 20 to 40% of patients, 3-cm margins in 9 to 15.5% of patients, and 5-cm margins in 5% of patients. Peripheral margins of 5 cm have a close to 0% recurrence rate. Wide resections may also not be practical in patients with tumors located in critical areas like the head and neck, and intraoperative frozen section assessments have not been reliable for determining margin status. Deep margins should always include excision of the deep fascia. The complete resection of the tumor requires excision of the external outer table in the cranium; muscle in trunk and extremities; peritoneum in thin patients with DFSP located on the abdomen, sternum, and clavicles; and vertebral apophysis when located on the thorax.

2020

41
Q

Cutaneous squamous cell carcinoma arises from which of the following epidermal layers?

A) Stratum basale
B) Stratum corneum
C) Stratum granulosum
D) Stratum lucidum
E) Stratum spinosum

A

The correct response is Option A.

Squamous cell carcinoma arises from keratinocytes in the stratum basale. The epidermis comprises four layers, from deep to superficial: stratum basale, stratum spinosum, stratum granulosum, and stratum corneum. Glabrous skin has an additional layer between the stratum granulosum and stratum corneum called stratum lucidum. The stratum basale consists of basal cells. These are stem cells that differentiate into keratinocytes. These keratinocytes then migrate upwards. In the stratum spinosum the keratinocytes form intercellular connections via desmosomes. In the stratum granulosum, the keratinocytes have keratohyalin granules. In the stratum corneum, the cells are compact and surrounded by a lipid layer, thus giving the skin its barrier function. Melanocytes and Merkel cells are found in the stratum basale. Langerhans cells are found in the stratum spinosum, stratum granulosum, and the dermis.

2020

42
Q

A 43-year-old man presents with a recurrent 6-mm skin lesion on his upper lip. It is characterized by an erythematous papule surrounded by scale and crust. Biopsy shows nests of poorly differentiated cords of spindle cells of keratinocyte origin. Which of the following is the most appropriate treatment?

A) Cryosurgery
B) Electrodessication and curettage
C) Mohs micrographic surgery
D) Radiation therapy
E) Standard surgical excision

A

The correct response is Option C.

The patient’s histology is consistent with squamous cell carcinoma. Mohs micrographic surgery (MMS) has the highest cure rates for both primary and recurrent squamous cell carcinoma. For locally recurrent tumors, the 5-year cure rate for MMS is 90% compared with 76.7% for standard surgical excision. Because MMS is a tissue-sparing technique, smaller surgical margins are taken, and scarring and functional impairment are minimized compared with standard surgical excision and electrodessication and curettage. Tumor removal and reconstruction are usually performed on the same day, using local anesthesia in an office-based setting. Recurrence rates after cryosurgery and radiation therapy are not as favorable as for MMS.

2020

43
Q

A 12-year-old boy presents with warts on his fingers. His parents report that the warts have persisted despite application of a variety of topical treatments. Which of the following viruses is the most likely cause of this patient’s condition?

A) H1N1
B) Hepatitis C
C) Herpes simplex
D) Human immunodeficiency
E) Human papilloma

A

The correct response is Option E.

The cause of warts is the human papilloma virus (HPV). There are more than 100 known types of HPV. Types 2 and 4 are the most common cause of warts on the hands, as in this patient. People whose occupations expose their hands to wet environments, such as meat, poultry, and fish handlers and veterinary surgeons have higher risk of developing warts. The virus can be transmitted on shared clothing or public spaces, such as showers, and then gain entry through breaks in the skin. The virus is then able to alter the squamous epithelium to produce a cauliflower-like growth. Warts can be present in single lesions or multiple lesions. They are often painless and are usually not cancerous but can be a source of embarrassment for the patient. Warts are generally self-limited and will resolve spontaneously within months or years. Spontaneous resolution appears to occur in 50% of children within 1 year and 90% within 5 years.

There are many treatments for warts. Home remedies include topical salicylic acid, duct tape, and cold treatments. Clinical treatments have shown topical acids and cryotherapy to be equally effective. Intralesional injections and topical antivirals, as well as immunologic stimulators of interferon, have been tried with some success, including purified candida; measles, mumps, and rubella; and tuberculin (PPD) protein. Chemical ablation with silver nitrate has been shown to resolve almost half of warts a month after a 9-day treatment protocol. Mechanical removal can be performed with direct excision with good success. Pulsed dye and CO2 lasers have been successfully used to ablate warts but can be painful, expensive, and leave scars. Periungal warts can be particularly difficult to treat topically and may require a more invasive treatment method. A patient’s ability to eradicate a wart can be reduced by a compromised immune system. Multiple progressive warts in immunosuppressed patients may need to be biopsied as these warts may transform into squamous cell carcinoma.

Human immunodeficiency virus (HIV), H1N1, hepatitis C, and herpes simplex are all viruses that affect humans but do not cause lesions on the fingers. Herpes simplex virus is the cause of cold sores around the mouth and genital herpes. Hepatitis C is a virus that causes inflammation of the liver. HIV decreases the effectiveness of the immune system. H1N1 is an influenza virus and known cause of the “swine flu” outbreak in 2009.

2020

44
Q

A patient undergoes excision of a low-risk basal cell carcinoma lesion that is 1.5 cm in diameter. According to recommended guidelines, which of the following surgical margins is most appropriate to excise?

A) 1 mm
B) 2 mm
C) 4 mm
D) 6 mm
E) 8 mm

A

The correct response is Option C.

Basal cell carcinoma (BCC) is the most common cancer in the United States and it is estimated that BCC occurs in close to 2 million Americans annually. This exceeds the incidence of all other cancers combined. Fortunately, the mortality rate is low, but there is certainly significant morbidity because the incidence of this common malignancy is increasing rapidly. BCC is twice as common as squamous cell carcinoma (SCC), which is the second most common type of skin cancer.

The clinical margins chosen by the panel for low risk tumors are based on the work of Wolf and Zitelli. Many other authors and consensus panels have agreed with their recommendation. Higher risk lesions and clinical signs may make it appropriate to increase or decrease this margin control based upon anatomic and systemic considerations.

2020

45
Q

A 53-year-old man presents with the lower eyelid skin lesions shown in the photograph. The lesions have been slowly growing over the past 12 months. Which of the following treatment options is the most appropriate?

A) Excision of the mass and overlying skin
B) Excision with 1-mm tissue margins
C) Excision with 2-mm tissue margins
D) Excision with 4-mm tissue margins and sentinel lymph node biopsy
E) Mohs micrographic surgery

A

The correct response is Option A.

The patient presents with xanthelasmata, which are localized accumulation of lipid deposits on the eyelids. Multiple treatment modalities are available, including chemical peels, cryotherapy, and laser ablation. Traditionally, surgical excision has been used and yields excellent cosmetic outcomes. As the lesions are benign, there is no indication for Mohs micrographic surgery and no margins are required.

2021

46
Q

A 32-year-old woman is referred by her dermatologist for evaluation and treatment of a painful skin lesion on her lateral cheek. Examination shows a 5-mm raised skin lesion with a bluish hue. Excisional biopsy with 1-mm margins is performed. Pathologic analysis shows a well-circumscribed tumor arising within the dermis with diffuse dense basophilic cellular proliferation, eosinophilic hyaline deposits, and a lymphocytic infiltrate. The pathologic diagnosis is spiradenoma. Which of the following is the most appropriate next step in management of this lesion?

A) Chemotherapy targeting lymphocytes
B) Radiation therapy
C) Re-excision with 1-cm margins
D) Sentinel lymph node biopsy
E) Reassurance and observation

A

The correct response is Option E.

The most appropriate next step in management of a completely excised spiradenoma is reassurance and observation. Spiradenomas are well-differentiated, benign, dermal neoplasms. Their origin is controversial, with some believing that they originate from sweat glands and others believing that they originate from hair follicles. Most spiradenomas are seen in patients between 15 to 35 years of age. They usually present as small, solitary, painful nodules that can grow to several centimeters, often with a bluish hue. Spiradenomas usually arise on the head, neck, and trunk. There are various morphological subtypes, and they can occur alongside cylindromas, trichoepitheliomas, and/or trichoblastomas. Patients with Brooke-Spiegler syndrome have multiple spiradenomas, cylindromas, and trichoepitheliomas.

2021

47
Q

A 65-year-old man presents with the rapidly growing, painful dorsal hand lesion shown in the photographs. It is not fixed to the underlying structures and the tendon and bone are not involved. He is a kidney transplant recipient and has been on immunosuppression for the past 10 years. Eighteen months ago, he was treated for a previous squamous cell carcinoma on the contralateral hand. Which of the following are the most appropriate surgical margins and depth of excision for this lesion?

A) 1-cm peripheral margins, excision to the deep dermal level
B) 1-cm peripheral margins, excision to the deep subcutaneous level
C) 1-mm peripheral margins, excision to the deep dermal level
D) 1-mm peripheral margins, excision to the deep subcutaneous level

A

The correct response is Option B.

Squamous cell carcinoma that arises in immunocompromised patients tends to behave more aggressively than tumors in immunocompetent patients. Resection with at least 6-mm margins (some recommend up to 10-mm margins), extending into the subcutaneous tissue, is recommended. Cure rates with Mohs surgery and with frozen section margin control are similar to those for wide local excision.

One-mm surgical margins are too narrow for squamous cell carcinoma, and would lead to a high rate of reoperations for positive microscopic margins. Furthermore, the entire dermis must be completely excised, along with at least part of the subcutaneous tissue, as lesions like this are likely to penetrate to that depth.

2021

48
Q

A 55-year-old woman undergoes biopsy of a suspicious lesion on the dorsal hand. Which of the following is the most common malignant skin tumor of the hand?

A) Basal cell carcinoma
B) Keratoacanthoma
C) Melanoma
D) Merkel cell carcinoma
E) Squamous cell carcinoma

A

The correct response is Option E.

Malignant tumors are uncommon in the hand, and squamous cell cancer is by far the most common malignant tumor. Approximately 75% of malignant skin lesions on the hand are squamous cell cancer and appear as crusty, scaly raised lesions predominantly on the dorsal skin. They will occasionally ulcerate. Basal cell cancer is a common malignant skin tumor but only accounts for about 10% of hand skin cancers. Melanoma is unusual on the hand; it is responsible for about 3% of hand tumors. Merkel cell carcinoma of the hand is rarely found. Keratoacanthoma is a mimicker of squamous cell carcinoma with a much more rapid growth rate and spontaneous remission.

2021

49
Q

A 7-year-old girl is evaluated because of a 1.5-cm, slow-growing, isolated, firm subcutaneous mass on the posterolateral neck that has been present for the past year. Examination of a specimen obtained on excisional biopsy results in a diagnosis of pilomatrixoma. The specimen is most likely to have a mutation of which of the following genes?

A) CTNNB1
B) GLUT1
C) NF1
D) p57

A

The correct response is Option A.

Pilomatrixoma (also known as pilomatricoma or calcifying epithelioma of Malherbe) is a benign, slow-growing skin tumor of the hair follicle. These tumors are most commonly found in children, although they have been increasingly found in patients of all ages. They tend to develop in the head and neck region but can also be found in the trunk and extremities, and they are usually not associated with any other isolated signs and symptoms. Pilomatrixomas can rarely become cancerous via transformation to the malignant pilomatrix carcinoma. Mutations in the CTNNB1 gene have been found in at least 75% of isolated pilomatrixomas. The CTNNB1 gene is needed to regulate cell growth and attachment, and mutation in this gene directly implicates beta-catenin/LEF dysregulation as the major cause of hair matrix cell tumorigenesis in this condition.

The GLUT1 gene mutation is associated with infantile hemangioma, while the NF1 and p57 gene mutations are associated with neurofibromatosis 1 and Beckwith-Wiedemann syndrome, respectively.

2021

50
Q

Which of the following structures contains apocrine sweat glands?

A) Areola
B) Glans penis
C) Labia minora
D) Palms of the hands
E) Vermilion border of the lips

A

The correct response is Option A.

Sweat glands are of three types: eccrine, apocrine, and apoeccrine. Apocrine glands are located in the areola, axilla, labia majora, scrotum, prepuce, periumbilical, and perianal areas. Their ducts deposit sweat into hair follicles. Apocrine glands start functioning at puberty and secrete a fluid that contains several compounds, some of which are broken down by bacteria to generate odor. Their secretions also contain pheromones. Apocrine glands are thought to be involved in the pathogenesis of hidradenitis suppurativa.

Eccrine sweat glands are distributed throughout the body, except the vermilion border of the lip, nail bed, external auditory canal, nipples, labia minora, glans penis, and clitoris. The palms and soles have the highest density of eccrine sweat glands in the body. Their ducts deposit sweat directly onto the skin surface. This sweat is a clear, colorless, and odorless fluid that contains water and electrolytes. Their main function is temperature homeostasis. They also participate in the barrier function of skin by secreting immunoglobulins. Eccrine glands also contain stem cells that play an important role in skin repair and wound epithelialization. Abnormally increased activity of these glands in response to emotional stimuli results in hyperhidrosis.

Apoeccrine glands share histologic features of both apocrine and eccrine glands and present in the axilla and perianal areas. They secrete watery sweat in response to psychological stress. High numbers of these glands are thought be an etiologic factor in patients with axillary hyperhidrosis.

2022

51
Q

A 43-year-old African American woman presents after undergoing Wise pattern reduction mammaplasty 6 months ago. Examination shows bilateral medial inframammary scars that are raised and pruritic but have not extended beyond the original incision sites. Intralesional injection of triamcinolone (TAC) is planned. Which of the following drugs, when administered in combination with TAC, will improve long-term efficacy?

A) 5-Fluorouracil
B) Diphenhydramine
C) Methotrexate
D) Tamoxifen
E) Verapamil

A

The correct response is Option A.

This patient has hypertrophic scars. Injection of 5-fluorouracil (5-FU) and TAC has been shown in randomized trials to be superior to TAC alone in the treatment of hypertrophic scars. Studies have used a regimen of 4 mg TAC and 45 mg 5?FU once a week for 8 weeks and showed reductions in Patient and Observer Scar Assessment Scales. In low doses, methotrexate (MTX) has anti-inflammatory effects mediated by adenosine A2 receptors. Oral MTX has shown promise in preventing recurrence of keloids after excision, but no definitive randomized data yet exist. Verapamil has been shown to be inferior to intralesional TAC in a randomized trial. Tamoxifen and diphenhydramine have mechanistic interest in the treatment of hypertrophic scars, but no data currently exist.

2022

52
Q

An 80-year-old man presents with a primary basal cell carcinoma of the left upper lip. Which of the following factors is associated with the highest risk for recurrence?

A) Desmoplastic subtype
B) Perineural involvement
C) Poor differentiation
D) Primary lesion
E) Well-circumscribed borders

A

The correct response is Option B.

Perineural involvement is the factor most associated with high risk for recurrence of basal cell carcinoma.

Poorly defined borders, a recurrent lesion, an immunosuppressed patient, the site of prior radiotherapy, perineural involvement, and an aggressive histologic subtype (morpheaform, basosquamous, sclerosing, mixed infiltrative, or micronodular) are the factors associated with a high risk for recurrence of basal cell carcinoma.

Poorly defined borders, recurrent disease, an immunosuppressed patient, the site of prior radiotherapy or chronic inflammation, a rapidly growing tumor, neurologic symptoms, perineural or vascular involvement, poor differentiation, or adenoid, adenosquamous, or desmoplastic subtypes are the factors associated with a high risk for recurrence of squamous cell carcinoma.

2022

53
Q

A 67-year-old retired landscaper with Fitzpatrick type I skin has a 5-year history of numerous flat erythematous scaly skin lesions (>20) on his bald scalp. He reports several new skin lesions of similar appearance on routine follow-up. Examination of several specimens obtained on shave biopsy shows cellular atypia above the basement membrane zone with overlying parakeratosis. He has no prior history of skin malignancy. Which of the following is the most appropriate initial treatment?

A) Cryotherapy
B) Systemic pembrolizumab
C) Topical 5-fluorouracil
D) Wide local excision
E) Observation

A

The correct response is Option C.

Actinic keratosis is a pre-malignant skin lesion that occurs on sun-exposed skin. Risk factors for actinic keratosis include advanced age, male sex, cumulative sun exposure, and fair skin type. The risk of progression from actinic keratoses to invasive squamous cell carcinoma is 0.025 to 16% per year, and the calculated lifetime risk for progression across a 10-year follow-up is approximately 6 to 10%.

The goals of the treatment are to eliminate clinical and subclinical actinic keratoses, minimize their risk for malignant transformation to invasive squamous cell carcinoma, and obtain acceptable cosmetic outcomes.

Topical field-directed therapy is the most appropriate initial next step in treatment of multiple, widespread actinic keratoses within an area of chronic sun damage. Topical medications include 5-fluorouracil (5-FU), imiquimod, diclofenac sodium, and ingenol mebutate. 5-Fluorouracil is an FDA-approved topical chemotherapeutic pyrimidine analog to treat multiple undetectable and clinically detectable actinic keratosis, and it is associated with a clearance rate of 84%. The mechanism of action is interference of DNA synthesis by inhibiting thymidylate synthetase. The one to two times per day application for 2 to 4 weeks is well tolerated with local skin reactions: pain, pruritus, burning, erythema, erosion, and inflammation.

Despite ongoing observation with routine skin examinations, the patient presented with additional actinic skin lesions. While observation may offer a nontreatment option, actinic skin lesions may undergo malignant change over time. Therefore, topical field-directed therapy is the best initial treatment option for widespread actinic keratoses.

Cryotherapy is a lesion-directed treatment option for a few individual actinic keratoses. It is a destructive modality often using liquid nitrogen (?195°C or ?319°F) that is better tolerated when less than 15 lesions are present. Clearance rates range from 39 to 99.8% with the use of cryotherapy. Common adverse effects include mild discomfort, scarring, and dyschromia.

Wide local excision is a directed therapy for individual malignant lesions, and it is therefore not appropriate in this patient due to the extensive nature of this benign condition.

Pembrolizumab is a systemic immune checkpoint inhibitor of PD-1 that treats metastatic or unresectable cutaneous melanoma.

2022

54
Q

A 36-year-old woman, gravida 1, para 1, with a history of cesarean delivery is evaluated for a painless, firm, 6-cm wide subcutaneous mass that is fixed to the anterior abdominal wall with no associated symptoms. The mass was noted after pregnancy. Tumor markers are within normal limits. She has a family history of familial adenomatous polyposis (FAP) syndrome. Which of the following tumors is the most likely diagnosis?

A) Dermatofibrosarcoma protuberans
B) Desmoid
C) Lipoma
D) Lymphoma
E) Neurofibroma

A

The correct response is Option B.

Abdominal wall tumors are rare, accounting for less than 10% of all soft-tissue tumors. Desmoid tumors and soft-tissue sarcomas account for 45% and 40%, respectively, of all abdominal wall tumors. Desmoid tumors, also known as aggressive fibromatosis, are characterized by unpredictable progression or spontaneous regression, but lack the ability to metastasize. The majority of desmoid tumors arise from sporadic mutations in CTNNB1, whereas 10% arise in association with an APC mutation in familial adenomatous polyposis (FAP) syndrome. Abdominal wall desmoid tumors demonstrate an increased prevalence in women of childbearing age.

Dermatofibrosarcoma protuberans (DFSP) is an uncommon, locally aggressive abdominal wall tumor with low metastatic potential. DFSP originates from cutaneous tissues and is limited to superficial structures. The majority of abdominal wall DFSP occur in adults aged 20 to 50 years with similar sex distribution, and tumors are small (<5 cm) with characteristic purple or blue discoloration.

Neurofibromas appear as soft, skin-colored papules or small, subcutaneous nodules. The majority of neurofibromas are localized and arise from sporadic neurofibromin 1 gene (NF1). While the plexiform type is pathognomonic for hereditary neurofibromatoses caused by germline mutations in the neurofibromin 1 gene (NF1) or neurofibromin 2 gene (NF2).

Lipomas are common benign tumors composed of mature adipose cells. Lipomas usually develop on the trunk or proximal limbs as discrete rubbery masses in the subcutaneous tissues that present at any age. Among solitary cutaneous lipomas, 60% display clonal alterations, which are not associated with presentation of multiple lipomas. There is an increased prevalence of solitary lipomas in women, and multiple lipomas occur more frequently in men.

Primary abdominal wall lymphoma is a rare extra-nodal presentation with increased prevalence among male patients. Extra-nodal disease may present at all ages, but more that 75% of patients are over age 50 years. Patients with atypical lymphoma may not present with fever, night sweat, weight loss or anemia. Lactate dehydrogenase tumor marker is observed in extra-nodal lymphoma. Most soft-tissue lymphomas are of B-cell origin.

2022

55
Q

A 75-year-old man presents with an abnormal skin lesion. Biopsy confirms Merkel cell carcinoma. Which of the following best describes the characteristics of this skin cancer?

A) The behavior is aggressive with high rates of lymph node spread and local recurrence
B) The plantar surface of the foot is the most common location for this lesion
C) This disease is more common amongst patients of African descent
D) This lesion frequently arises within the setting of a chronic wound
E) Topical chemotherapeutic agents are the primary method for managing this condition

A

The correct response is Option A.

Merkel cell carcinoma (MCC) is an uncommon skin cancer which has aggressive behavior. MCC usually presents as a painless red nodule that is rapidly growing. Elderly patients are more commonly affected by MCC, and it is believed that sun exposure is a major risk factor. There are several studies that suggest involvement of the polyomavirus in the pathogenesis of MCC. Patients with immunosuppression are also at relatively higher risk for MCC. This is an aggressive skin cancer with early and frequent involvement of the regional lymph nodes. Patients with large tumors or regional spread are at high risk for distant metastasis. This disease is significantly more common in lighter-skinned patients than in dark-skinned patients. The head and neck region is the most common site for MCC. A Marjolin’s ulcer is a rare and aggressive type of squamous cell carcinoma arising in the setting of a chronic wound. Once a diagnosis has been established, treatment of MCC typically involves wide local excision and lymph node mapping. Many of the treatment strategies for MCC are similar to melanoma given the similar aggressive clinical behavior.

2022

56
Q

A 35-year-old African American woman presents with multiple draining sinus tracts and nodular abscesses in the bilateral inframammary folds, groins, and axillae. She has failed topical and oral antibiotic therapies. Medical history includes type 2 diabetes mellitus, obesity, and keloid scarring. Which of the following is the most appropriate initial treatment for this patient?

A) Adalimumab therapy
B) Incision and drainage
C) Radiotherapy
D) Skin-tissue-saving excision with electrosurgical peeling (STEEP) procedure
E) Wide excision and skin grafting

A

The correct response is Option A.

This patient has hidradenitis suppurativa characterized by multiple nodules, abscesses, tunnels, and scars most commonly in the axillae, inframammary folds, groin, perigenital, and perineal region. Significant advances in medical therapy have decreased the need for surgical intervention. The disease is classified according to the Hurley classification: stage I as transient nonscarring inflammatory lesions; stage II as separate lesions consisting of recurrent abscesses with tunnel formation and scarring, and single or multiple lesions separated by normal-looking skin; and stage III as coalescent lesions with tunnel formation, scarring and inflammation. This patient is stage II. Mild disease is often treated with topical antibiotics (e.g., clindamycin) and/or resorcinol, while moderate disease may benefit from oral antibiotics (e.g., tetracycline 500 mg twice daily). Refractory disease may benefit from antibody therapy and/or surgical intervention.

Adalimumab (Humira) is a recombinant human igG1 anti-TNF monoclonal antibody that binds the proinflammatory cytokine TNF-alpha. It was approved by the FDA in 2016 for the treatment of moderate to severe disease where patients have required long-term antibiotics or rapid flares upon cessation of antibiotics. Alternative agents include infliximab (Remicade), anakinra (Kineret), ustekinumab (Stelara), dapsone, or acitretin (Soriatane).

Radiotherapy is incorrect. There are no randomized trials comparing radiotherapy with medical or surgical therapy, but it can be effective and well-tolerated for focal areas such as the scalp. The risk for secondary cancer is minimal but not negligible; therefore, radiotherapy is not considered a front-line option.

Surgical incision and drainage is indicated for fluctuant abscesses but not effective for inflamed nodules since there is no collection of fluid to drain.

Skin-tissue-saving excision with electrosurgical peeling (STEEP) or deroofing removes unhealthy tissue with step-wise tangential excisions and preserving normal tissue to heal via secondary intention. There is significant postoperative morbidity and risk for scarring with higher recurrence rates. Given this patient’s history for keloid scarring, this is not the best option.

Wide excision is associated with lower recurrence rates but higher morbidity (e.g., infection, bleeding, contractures). This can be effective in patients with areas of limited disease but should be reserved for patients with severe disease refractory to nonsurgical therapies when large total body surface area is involved. Nonsurgical options should be fully explored in this patient given the keloid history.

2022

57
Q

A 79-year-old farmer presents with a lesion on the back of his neck. Examination shows the lesion is flat and pink and rough to the touch. Which of the following is the estimated rate of malignant transformation for an actinic keratosis?

A) 1%
B) 10%
C) 25%
D) 50%
E) 75%

A

The correct response is Option B.

While the true rate of malignant transformation of actinic keratosis remains unknown, best estimates report the progression rate to squamous cell or basal cell carcinoma is 10%. As a result, treatment for these lesions is recommended through a variety of modalities. The most commonly used method of treatment for patients with solitary or few lesions is cryotherapy. Other options include curettage and topical therapies. For multiple lesions of field cancerization of an anatomical area, topical fluorouracil (5-FU), imiquimod, or photodynamic therapy with photosensitizers are more often used.

2022

58
Q

A 70-year-old man presents with a 2-cm keratoacanthoma involving the left ear. Medical history includes melanoma of the left cheek. Which of the following is an indication for Mohs micrographic surgery over conventional excision in this patient?

A) Anatomic location of lesion
B) Diagnosis of keratoacanthoma
C) History of previous melanoma
D) Male biological sex
E) Patient age

A

The correct response is Option A.

Mohs micrographic surgical technique has demonstrated cure rates of 97% for primary squamous cell carcinomas and up to 95% for recurrent squamous carcinomas. In this particular patient, the strongest indication for utilization of the Mohs technique is the involvement of the patient’s ear. Keratoacanthomas are epithelioid neoplasms of sun-exposed skin. Classically, they appear as a solitary papule that rapidly increases in size with a crateriform center. Because they can undergo spontaneous involution, some groups feel keratoacanthomas should be classified as a benign growth. However, there is strong evidence that keratoacanthomas are a distinct variant of squamous cell carcinoma. They have been reported to demonstrate invasion and aggressive behavior, including perineural invasion in up to 2.5 to 14% in the head and neck. Because squamous cell carcinoma involvement of the ear increases the risk for recurrence and metastasis, Mohs micrographic surgery is indicated in this patient. Patient age, patient sex, history of previous melanoma, and the diagnosis of keratoacanthoma are not indications for Mohs excision.

Other indications for Mohs technique include the following:

  1. Recurrent basal cell/squamous cell carcinomas
  2. Locations prone to recurrence—”H-zone” of the face: periorbital, periauricular, temple, upper lip, nose/nasolabial fold, and chin
  3. Tumors involving critical structures such as the eyelid or lip
  4. Functionally important areas such as the genitals, perianal location, hands, and feet
  5. Tumors arising in sites of previous irradiation therapy
  6. Large tumors, >2 cm
  7. Lesions with ill-defined tumor margins
  8. Histologic aggressive subtype (morpheaform, basosquamous, perineural, and invasive/poorly differentiated squamous cell carcinoma)
  9. Tumors arising in immunosuppressed patients, such as transplant recipients or patients with genetic predisposition (basal cell nevus syndrome, xeroderma pigmentosum)

2022

59
Q

A 55-year-old man presents with a fungating mass of the right shoulder that has been enlarging for the past 12 years. A photograph is shown. Medical history is unremarkable. The patient reports that he spent his youth working outside. He has a history of smoking. Which of the following is the most significant risk factor for development of this lesion in this patient?

A) CDK4 mutation
B) Previous use of topical steroids
C) Smoking history
D) Sun exposure

A

The correct response is Option D.

The patient presents with a nodular ulcerated basal cell carcinoma to the right shoulder. Basal cell carcinoma is the most common malignancy in the United States due to the increase in sun exposure and tanning salons. The most common risk factor for basal cell carcinomas is sun exposure. Generally, they do not metastasize and are resectable, but do lead to large oncologic resections if left to progress, as is the case in the patient described. Smoking history, although important, does not have a major impact on the risk for basal cell cancer. Marjolin ulcers are a variant of squamous cell carcinoma that results from the chronic inflammatory process that follows burns and is not a risk factor for basal cell carcinoma. Immunosuppression is a risk factor for basal cell carcinomas, but it is not as great as sun exposure and this patient has no history of immunosuppression. Other risk factors for basal cell carcinoma include being male and of older age, but these are not listed as possible choices. A mutation in the CDK4 gene is linked to familial melanoma and would have no relation to the basal cell tumor in this patient. Topical steroid use has no known impact on risk for development of basal cell carcinoma.

2022

60
Q

A 60-year-old woman presents with worsening painful ulceration of a prior healed surgical site of the left lower extremity after undergoing hidradenitis excisions 10 years ago. She reports no trauma and has had no fevers or chills. Examination shows some drainage at the wound site. She undergoes treatment with debridement, antibiotics, and multiple types of dressings without progress of wound healing. Photographs are shown. Which of the following is the most appropriate next step in management?

A) Debridement and biopsy
B) Debridement and dermal allografting
C) Debridement and negative pressure dressing
D) Excision and coverage with a flap
E) Excision and skin grafting

A

The correct response is Option A.

Marjolin ulcers are malignancies (most commonly aggressive squamous cell carcinoma) that arise from areas of chronic irritation, injury, or unstable scar. These malignancies frequently occur over many years. This malignant degeneration is most commonly associated with burns; however, it can be associated with a multitude of other types of nonhealing wounds such as traumatic wounds, osteomyelitis, pressure sores, any type of ulcer, laceration, venous stasis ulcer, or fistulas. The rate of incidence is approximately 2% in burn scars, with a predilection for the extremities, especially in the lower limbs. There is often a delay in diagnosis, since the appearance of a nonhealing ulcer with heaped-up edges mimics other types of ulcerations. Marjolin ulcer should be ruled out in a chronic nonhealing ulcer in the setting of a long-term scar/injury/burn.

Diagnosis of Marjolin ulcer is typically made by tissue biopsy with pathologic evaluation. Squamous cell carcinoma is the most common type of histology seen in Marjolin ulcer, although basal cell carcinoma, malignant fibrous histiocytoma, malignant melanoma, liposarcoma, fibrosarcoma, eccrine syringofibroadenoma, Merkel cell carcinoma, and keratoacanthoma have all been described. For patients with squamous cell carcinoma (most common type), the degree of differentiation is important prognostically. The risk for metastasis is highly correlated with the degree of differentiation and grade of the tumor. There is early lymph node metastasis in 30% of patients.

Treatment is surgical with wide local excision with pursuit of negative margins. Sentinel lymph node biopsy is frequently pursued. Deeper ulcers involving bone may warrant amputation of the involved extremity.

Marjolin ulcer must be diagnosed and treated (or the diagnosis excluded) before considering any other treatments.

2022

61
Q

A 50-year-old woman presents with a 1.5-cm white plaque on the left cheek that has been slowly growing for the past 3 years. A photograph is shown. Examination of the specimen obtained on biopsy shows perineural invasion and basaloid epitheloid cells displaying ductal lumina. Which of the following is the most likely diagnosis?

A) Keratoacanthoma
B) Malignant melanoma
C) Microcystic adnexal carcinoma
D) Squamous cell carcinoma

A

The correct response is Option C.

Microcystic adnexal carcinoma (MAC) is a slow-growing skin cancer that primarily affects the head and neck and usually presents as a fleshy, plaque-like lesion. MAC is commonly misdiagnosed as basal or squamous cell carcinomas amongst others. MAC commonly has perineural invasion (PNI) and, as its names implies, affects apocrine structures. Although PNI can occur with basal cell carcinoma, squamous cell carcinoma, and melanoma, the latter two are generally much faster growing and this finding is still much less common. For basal cell carcinoma, PNI is typically observed in patients with advanced disease. Keratoacanthoma is a form of well-differentiated squamous cell carcinoma and is generally fast growing. Treatment of MAC is typically carried out with complete circumferential, peripheral, and deep margin assessment, and most often with Mohs micrographic surgery.

2022

62
Q

A 17-year-old girl presents to the clinic to have the lesion in the photograph removed. She has had it since birth. She does not like its appearance and has noted it has changed in size and shape during the last two years. Histopathologic examination of the specimen is most likely to show which of the following?

A) Amelanotic melanoma
B) Basal cell carcinoma
C) Nevus sebaceous
D) Squamous cell carcinoma
E) Trichoblastoma

A

The correct response is Option C.

The photograph shows a sebaceous nevus of Jadassohn, and on pathology, a benign organoid nevus is the most likely finding. Given the patient’s age, malignant transformation is very rare (around 2.5%). Until recently, the most common transformation has been thought to be basal cell carcinoma (1.1%). However, it is now thought that many of these were trichoblastomas. A melanoma present since childhood does not fit the presentation of this patient.

2022

63
Q

A 63-year-old woman with a history of ulcerative colitis presents after being diagnosed with invasive ductal carcinoma of the right breast. Her BMI is 23 kg/m2. She undergoes unilateral mastectomy with immediate reconstruction with a free deep inferior epigastric perforator flap. Her initial postoperative course is uneventful. Five weeks after the procedure, the patient returns to the clinic. Examination shows erythema and skin ulcers surrounded by violaceous discoloration around the abdominal and breast incisions. No underlying fluid collections are appreciated. Punch biopsies of the ulcers are performed and show neutrophilic dermatosis. Which of the following is the most appropriate next step in management?

A) Broad-spectrum antibiotic therapy
B) CT scan of the chest/abdomen/pelvis
C) High-dose steroid therapy
D) Surgical exploration with washout and debridement
E) Ultrasound of the breast and abdomen

A

The correct response is Option C.

Pyoderma gangrenosum (PG) is a rare skin disorder of unknown cause that is believed to be part of the spectrum of neutrophilic dermatoses. Specific characteristics of postoperative PG include dramatic deterioration of surgical wounds after a period of 4 days to 6 weeks, with predominant involvement of the breast and abdomen. Numerous reports of PG following reduction mammaplasty and breast reconstruction have been published. Additionally, patients with a history of inflammatory bowel disease are at an increased risk for developing PG.

Clinically, postoperative PG is characterized by fever, chills, cellulitis, and wound breakdown. The hallmark finding is a rapidly enlarging area of central skin ulceration surrounded by violaceous skin with irregular borders. Systemic signs of inflammation along with laboratory abnormalities (leukocytosis, hyponatremia, and hypoproteinemia) can accompany these findings. The diagnostic challenge of postoperative PG is related to its clinical resemblance to necrotizing infection and, hence, a high index of suspicion is required. However, the presence of neutrophilic dermatosis supports the diagnosis of PG. Treatment consists of immunosuppression and frequently consists of a combination of high-dose steroids and topical tacrolimus.

Additional imaging would not aid in management in this case. Broadening antibiotic coverage or surgical debridement would be indicated for treatment of infection. Local wound care does not address the underlying cause of the clinical condition and, hence, would not be the most appropriate next step.

2022

64
Q

Which of the following subtypes of basal cell carcinoma has the lowest risk for local recurrence?

A) Infiltrating
B) Micronodular
C) Morpheaform
D) Nodular
E) Sclerosing

A

The correct response is Option D.

Basal cell carcinoma (BCC) is the most common human malignancy as well as the most common malignant tumor of the skin. The incidence of BCC is increasing worldwide. The most significant etiological factors are believed to be ultraviolet light exposure and genetic predisposition. Therefore, an aging population and prolonged exposure to sunlight may explain the worldwide increasing incidence.

The great majority of BCCs are successfully treated and will not recur. However, it is important to know the high risk lesions and subtypes to understand when the recurrence rate may be higher. The morpheaform, sclerosing, infiltrating, micronodular, and metatypical subtypes are associated with higher risk for relapse. Anatomic locations on the trunk and limbs are considered low-risk areas, while the forehead, cheek, chin, scalp, and neck are intermediate-risk areas. The nose and perioral areas are high-risk areas. Size greater than 1 cm for head and neck tumors and greater than 2 cm in other body areas also predisposes to a higher recurrence risk.

2022

65
Q

A 52-year-old man presents with a 1-cm invasive cutaneous squamous cell carcinoma of the penis. Which of the following is the most appropriate treatment?

A) 5-Fluorouracil therapy
B) Mohs micrographic surgery
C) Penis transplantation
D) Radiation therapy
E) Total amputation

A

The correct response is Option B.

Mohs micrographic surgery involves the removal of cutaneous malignancies with concomitant circumferential histologic examination. This technique helps preserve normal tissue and has low rates of recurrence. It is most appropriately used for aggressive lesions, lesions at high risk for recurrence, and in areas where maximal preservation of normal tissue is critical for form or function, such as the “mask” areas of the face or other sensitive areas. Appropriate use criteria for Mohs micrographic surgery were developed by an ad hoc committee representing several dermatologic professional societies. This set of recommendations categorizes basal cell carcinoma, squamous cell carcinoma, lentigo maligna, and melanoma in situ as appropriate for Mohs micrographic surgery, of uncertain benefit, or as inappropriate, based on lesion characteristics and location. Squamous cell carcinoma on the genitalia, whether aggressive or nonaggressive, falls in the appropriate for Mohs micrographic surgery category. Furthermore, studies have shown excellent results with Mohs micrographic surgery of the penis, with low recurrence rates. Sexual and urinary function is better preserved by Mohs micrographic surgery than with traditional partial or total amputation.

5-Fluorouracil is a topical chemotherapeutic agent which may be used to treat actinic keratosis, superficial basal cell carcinoma, or squamous cell carcinoma in situ, but it is not appropriate in this patient. Radiation therapy may be useful for treatment of nonmelanoma skin cancer in patients for whom surgery is a poor option. However, when compared with Mohs micrographic surgery in this patient, radiation would likely be less effective and lead to increased morbidity. Total amputation of the penis may have a high cure rate but carries with it devastating morbidity. It is not an appropriate first-line treatment in this patient. Penis transplantation remains an experimental procedure and currently requires life-long systemic immunosuppression. It is not the correct answer.

2022